12 Questions (Ch. 8), 4 Questions (Ch. 1), 2 Questions (Ch. 3), 3 Questions (Ch. 4), 2 Questions (Ch. 5), 4 Questions (Ch. 6), 4 Questions (Ch. 12), 4 Questions (Ch. 11), 5 Questions (Ch. 10), 4 Questions (Ch. 12)

Ace your homework & exams now with Quizwiz!

22. Which analysis of maternal serum may predict chromosomal abnormalities in the fetus? a. Multiple-marker screening b. Lecithin/sphingomyelin (L/S) ratio c. Biophysical profile d. Type and crossmatch of maternal and fetal serum

a. Multiple-marker screening

In caring for the woman with disseminated intravascular coagulation (DIC), what order should the nurse anticipate? A) Administration of blood B) Preparation of the woman for invasive hemodynamic monitoring C) Restriction of intravascular fluids D) Administration of steroids

*(A) Administration of blood* Rationale: Primary medical management in all cases of DIC involves correction of the underlying cause, volume replacement, blood component therapy, optimization of oxygenation and perfusion status, and continued reassessment of laboratory parameters. Central monitoring would not be ordered initially in a woman with DIC because this can contribute to more areas of bleeding. Management of DIC includes volume replacement, not volume restriction. Steroids are not indicated for the management of DIC.

A woman with severe preeclampsia is being treated with an IV infusion of magnesium sulfate. This treatment is considered successful if: A) blood pressure is reduced to prepregnant baseline. B) seizures do not occur. C) deep tendon reflexes become hypotonic. D) diuresis reduces fluid retention.

*B) seizures do not occur.* Rationale: A temporary decrease in blood pressure can occur; however, this is not the purpose of administering this medication. Magnesium sulfate is a central nervous system (CNS) depressant given primarily to prevent seizures. Hypotonia is a sign of an excessive serum level of magnesium. It is critical that calcium gluconate be on hand to counteract the depressant effects of magnesium toxicity. Diuresis is not an expected outcome of magnesium sulfate administration.

The nurse is caring for a woman who is at 24 weeks of gestation with suspected severe preeclampsia. Which signs and symptoms would the nurse expect to observe? (Select all that apply.) A) Decreased urinary output and irritability B) Transient headache and +1 proteinuria C) Ankle clonus and epigastric pain D) Platelet count of less than 100,000/mm3 and visual problems E) Seizure activity and hypotension

*A) Decreased urinary output and irritability* *C) Ankle clonus and epigastric pain* *D) Platelet count of less than 100,000/mm3 and visual problems* Rationale: Decreased urinary output and irritability are signs of severe eclampsia. Ankle clonus and epigastric pain are signs of severe eclampsia. Platelet count of less than 100,000/mm3 and visual problems are signs of severe preeclampsia. A transient headache and +1 proteinuria are signs of preeclampsia and should be monitored. Seizure activity and hyperreflexia are signs of eclampsia.

A woman diagnosed with marginal placenta previa gave birth vaginally 15 minutes ago. At the present time she is at the greatest risk for: A) hemorrhage. B) infection. C) urinary retention. D) thrombophlebitis.

*A) hemorrhage.* Rationale: Hemorrhage is the most immediate risk because the lower uterine segment has limited ability to contract to reduce blood loss. Infection is a risk because of the location of the placental attachment site; however, it is not a priority concern at this time. Placenta previa poses no greater risk for urinary retention than with a normally implanted placenta. There is no greater risk for thrombophlebitis than with a normally implanted placenta.

A woman with severe preeclampsia has been receiving magnesium sulfate by IV infusion for 8 hours. The nurse assesses the woman and documents the following findings: temperature 37.1° C, pulse rate 96 beats/min, respiratory rate 24 breaths/min, blood pressure 155/112 mm Hg, 3+ deep tendon reflexes, and no ankle clonus. The nurse calls the physician, anticipating an order for: A) hydralazine. B) magnesium sulfate bolus . C) diazepam. D) calcium gluconate.

*A) hydralazine.* Rationale: Hydralazine is an antihypertensive commonly used to treat hypertension in severe preeclampsia. An additional bolus of magnesium sulfate may be ordered for increasing signs of central nervous system irritability related to severe preeclampsia (e.g., clonus) or if eclampsia develops. Diazepam sometimes is used to stop or shorten eclamptic seizures. Calcium gluconate is used as the antidote for magnesium sulfate toxicity. The client is not currently displaying any signs or symptoms of magnesium toxicity.

The emergency department nurse is assessing a pregnant trauma victim who just arrived at the hospital. What are the nurse's MOST appropriate actions? (Select all that apply.) A) Place the patient in a supine position. B) Assess for point of maximal impulse at fourth intercostal space. C) Collect urine for urinalysis and culture. D) Frequent vital sign monitoring. E) Assist with ambulation to decrease risk of thrombosis.

*B) Assess for point of maximal impulse at fourth intercostal space.* *C) Collect urine for urinalysis and culture.* *D) Frequent vital sign monitoring.* Rationale: Passive regurgitation may occur if patient is supine, leading to high risk for aspiration. Placental perfusion is decreased when the patient is in a supine position as well. The heart is displaced upward and to the left in pregnant patients. During pregnancy, there is dilation of the ureters and urethra, and the bladder is displaced forward placing the pregnant trauma patient at higher risk for urinary stasis, infection, and bladder trauma. The trauma patient can suffer blood loss and other complications, necessitating frequent monitoring of vital signs. While the pregnant patient is at risk for thrombus formation, the patient must be cleared by the health care provider before ambulating. The pregnant trauma patient is at higher risk for pelvic fracture, and therefore this condition must be ruled out first as well.

A pregnant woman presents to the emergency department complaining of persistent nausea and vomiting. She is diagnosed with hyperemesis gravidarum. The nurse should include which information when teaching about diet for hyperemesis? (Select all that apply.) A) Eat three larger meals a day. B) Eat a high-protein snack at bedtime. C) Ice cream may stay down better than other foods. D) Avoid ginger tea or sweet drinks. E) Eat what sounds good to you even if your meals are not well-balanced.

*B) Eat a high-protein snack at bedtime.* *C) Ice cream may stay down better than other foods.* *E) Eat what sounds good to you even if your meals are not well-balanced.* Rationale: The diet for hyperemesis includes: • Avoid an empty stomach. Eat frequently, at least every 2 to 3 hours. Separate liquids from solids and alternate every 2 to 3 hours. • Eat a high-protein snack at bedtime. • Eat dry, bland, low-fat, and high-protein foods. Cold foods may be better tolerated than those served at a warm temperature. • In general eat what sounds good to you rather than trying to balance your meals. • Follow the salty and sweet approach; even so-called junk foods are okay. • Eat protein after sweets. • Dairy products may stay down more easily than other foods. • If you vomit even when your stomach is empty, try sucking on a Popsicle. • Try ginger tea. Peel and finely dice a knuckle-sized piece of ginger and place it in a mug of boiling water. Steep for 5 to 8 minutes and add brown sugar to taste. • Try warm ginger ale (with sugar, not artificial sweetener) or water with a slice of lemon. • Drink liquids from a cup with a lid.

A woman with severe preeclampsia is receiving a magnesium sulfate infusion. The nurse becomes concerned after assessment when the woman exhibits: A) a sleepy, sedated affect. B) a respiratory rate of 10 breaths/min. C) deep tendon reflexes of 2+. D) absent ankle clonus.

*B) a respiratory rate of 10 breaths/min.* Rationale: Because magnesium sulfate is a central nervous system (CNS) depressant, the client will most likely become sedated when the infusion is initiated. A respiratory rate of 10 breaths/min indicates that the client is experiencing respiratory depression (bradypnea) from magnesium toxicity. Deep tendon reflexes of 2+ are a normal finding. Absent ankle clonus is a normal finding.

The most prevalent clinical manifestation of abruptio placentae (as opposed to placenta previa) is: A) bleeding. B) intense abdominal pain. C) uterine activity. D) cramping.

*B) intense abdominal pain.* Rationale: Bleeding may be present in varying degrees for both placental conditions. Pain is absent with placenta previa and may be agonizing with abruptio placentae. Uterine activity may be present with both placental conditions. Cramping is a form of uterine activity that may be present in both placental conditions.

The nurse is preparing to discharge a 30-year-old woman who has experienced a miscarriage at 10 weeks of gestation. Which statement by the woman would indicate a correct understanding of the discharge instructions? A) "I will not experience mood swings since I was only at 10 weeks of gestation." B) "I will avoid sexual intercourse for 6 weeks and pregnancy for 6 months." C) "I should eat foods that are high in iron and protein to help my body heal." D) "I should expect the bleeding to be heavy and bright red for at least 1 week."

*C) "I should eat foods that are high in iron and protein to help my body heal."* Rationale: After a miscarriage a woman may experience mood swings and depression from the reduction of hormones and the grieving process. Sexual intercourse should be avoided for 2 weeks or until the bleeding has stopped and should avoid pregnancy for 2 months. A woman who has experienced a miscarriage should be advised to eat foods that are high in iron and protein to help replenish her body after the loss. The woman should not experience bright red, heavy, profuse bleeding; this should be reported to the health care provider.

Signs of a threatened abortion (miscarriage) are noted in a woman at 8 weeks of gestation. What is an appropriate management approach for this type of abortion? A) Prepare the woman for a dilation and curettage (D&C). B) Place the woman on bed rest for at least 1 week and reevaluate. C) Prepare the woman for an ultrasound and blood work. D) Comfort the woman by telling her that if she loses this baby, she may attempt to get pregnant again in 1 month.

*C) Prepare the woman for an ultrasound and blood work.* Rationale: D&C is not considered until signs of the progress to an inevitable abortion are noted or the contents are expelled and incomplete. Bed rest is recommended for 48 hours initially. Repetitive transvaginal ultrasounds and measurement of human chorionic gonadotropin (hCG) and progesterone levels may be performed to determine if the fetus is alive and within the uterus. If the pregnancy is lost, the woman should be guided through the grieving process. Telling the client that she can get pregnant again soon is not a therapeutic response because it discounts the importance of this pregnancy.

A woman at 39 weeks of gestation with a history of preeclampsia is admitted to the labor and birth unit. She suddenly experiences increased contraction frequency of every 1 to 2 minutes; dark red vaginal bleeding; and a tense, painful abdomen. The nurse suspects the onset of: A) eclamptic seizure. B) rupture of the uterus. C) placenta previa. D) placental abruption.

*D) placental abruption.* Rationale: Eclamptic seizures are evidenced by the presence of generalized tonic-clonic convulsions. Uterine rupture presents as hypotonic uterine activity, signs of hypovolemia, and in many cases the absence of pain. Placenta previa presents with bright red, painless vaginal bleeding. Uterine tenderness in the presence of increasing tone may be the earliest finding of premature separation of the placenta (abruptio placentae or placental abruption). Women with hypertension are at increased risk for an abruption.

List the time span in lunar months, calendar months, weeks, and days that indicates the appropriate length for a normal pregnancy. ____________________

10 lunar months, 9 calendar months, 40 weeks, 280 days

7. What is the primary role of practicing nurses in the research process? a. Designing research studies b. Collecting data for other researchers c. Identifying researchable problems d. Seeking funding to support research studies

c. Identifying researchable problems

2 This discomfort of pregnancy usually subsides by the 20th week of pregnancy. An absolute definite end of vomiting during pregnancy can never be stated. Test-Taking Tip: Identifying content and what is being asked about that content is critical to your choosing the correct response. Be alert for words in the stem of the item that are the same or similar in nature to those in one or two of the options. Example: If the item relates to and identifies stroke rehabilitation as its focus and only one of the options contains the word stroke in relation to rehabilitation, you are safe in identifying this choice as the correct response.

A 24-year-old primipara, 10 weeks pregnant, who has been experiencing vomiting every morning for the past few weeks, asks the nurse at her check-up how long this "morning sickness" will continue. Which statement by the nurse is most accurate? 1 "It will end by the 15th week of pregnancy." 2 "It usually subsides by the 20th week of pregnancy." 3 "It's a very common but not serious problem." 4 "In some women, it can last throughout the pregnancy and become serious."

1, 2, 4, 5 Whenever a pregnant woman has nausea and vomiting, the first priority is a thorough assessment to determine the severity of the problem. In most cases the woman should be told to come immediately to the health care provider's office or the emergency department because the severity of the illness often is difficult to determine by telephone conversation. The assessment should include frequency, severity, and duration of episodes of nausea and vomiting. If the woman reports vomiting, the assessment also should include the approximate amount and color of the vomitus. The woman is asked to report any precipitating factors relating to the onset of her symptoms. Any pharmacologic or nonpharmacologic treatment measures used should be recorded. Prepregnancy weight and documented weight gain or loss during pregnancy are important to note. The woman's weight and vital signs are measured, and a complete physical examination is performed, with attention to signs of fluid and electrolyte imbalance and nutritional status. The most important initial laboratory test to be obtained is a determination of ketonuria. Other laboratory tests that may be prescribed are a urinalysis, a complete blood cell count, electrolytes, liver enzymes, and bilirubin levels. At this time, there is no supportive evidence that a blood transfusion is required. Based on provided objective data that the patient has deep furrows on her tongue, this may suggest a vitamin B deficiency which should be investigated further.

A 24-year-old primipara, who is 18 weeks pregnant, has been having increasing vomiting since she was 8 weeks pregnant. Upon arrival at the emergency department, her skin turgor is diminished, temperature is 99.2F (o), pulse is 102, respiration is 18, blood pressure is 102/68, and she has deep furrows on her tongue. What would the nurse expect to do to care for her? Select all that apply. 1 Start an intravenous infusion. 2 Check her urine for ketones 3 Cross match blood for a transfusion. 4 Obtain a complete history. 5 Obtain blood for a complete blood count

3 The nurse administers the prescribed Rho(D) immunoglobulin to the patient to protect the patient from isoimmunization. The nurse needs to obtain a prescription for magnesium sulfate if there are eclamptic seizures in a patient with preeclampsia. Oxytocin (Pitocin) is administered to prevent bleeding after birth or the evacuation of the uterus. Magnetic resonance imaging (MRI) is used to assess injuries in a patient after trauma.

A blunt abdominal trauma causes fetal hemorrhage in a pregnant patient. The nurse finds that the patient is Rh negative. What action does the nurse take? 1 Initiate magnesium sulfate per protocol. 2 Administer oxytocin (pitocin). 3 Administer prescribed Rho (D) immunoglobulin. 4 Prepare the patient for magnetic resonance imaging (MRI).

3 The graphic illustrates a depth of 6 mm when the nurse applies finger pressure to the swollen area. This measurement indicates a +3 measurement for edema. Upon applying finger pressure, a 2 mm depression would be the equivalent of a +1, a 4 mm depression, a +2, and an 8 mm depression a +4.

A labor and delivery nurse is in the process of admitting a patient who is 39 and at 5 weeks' gestation with a diagnosis of preeclampsia. The nurse has evaluated vital signs, weight, and deep tendon reflexes. Although the presence of edema is no longer included in the definition of preeclampsia, it is an important component of the nurse's evaluation. Edema is assessed for distribution, degree and pitting. Although the amount of edema is difficult to quantify, it is important to record the relative degrees of edema formation. From the graphic below, please select the illustration that best displays +3 edema. 1. A 2. B 3. C 4. D

4 Hyperemesis gravidarum is characterized by excessive vomiting during pregnancy, which causes nutritional deficiency and weight loss. The presence of ketonuria is another indication of this disorder. Preeclampsia refers to hypertension and proteinuria in patients after 20 weeks' gestation. Hyperthyroid disorder may be one of the causes of hyperemesis gravidarum. Gestational hypertension also develops after 20 weeks' gestation.

A patient reports excessive vomiting in the first trimester of the pregnancy, which has resulted in nutritional deficiency and weight loss. The urinalysis report of the patient indicates ketonuria. Which disorder does the patient have? 1Preeclampsia 2Hyperthyroid disorder 3Gestational hypertension 4Hyperemesis gravidarum

1 Labetalol hydrochloride (Normodyne) has a low concentration in breast milk, so the patient can breastfeed the infant. Breastfeeding is safe and will not cause convulsions or any side effects in the infant. Infant formulas are used only if the mother is unable to breastfeed the infant or if the mother does not desire to breastfeed.

A patient with gestational hypertension is prescribed labetalol hydrochloride (Normodyne) therapy, which is continued after giving birth. What does the nurse instruct the patient about breastfeeding? 1 "You may breastfeed the infant if you desire." 2 "Breastfeeding may cause convulsions in the infant." 3 "Breastfeed only once a day and use infant formulas." 4 "There may be high levels of the drug in the breast milk."

2 Systolic blood pressure exceeding 160 mm Hg indicates severe hypertension in the patient. The nurse should alert the health care provider and obtain a prescription for antihypertensive medications, such as nifedipine (Adalat) and labetalol hydrochloride (Normodyne). Magnesium sulfate would be administered if the patient was experiencing eclamptic seizures. Oral and intravenous fluids are restricted when the patient is at risk for pulmonary edema. Monitoring FHR and UCs is a priority when the patient experiences a trauma so that any complications can be addressed immediately.

A pregnant patient has a systolic blood pressure that exceeds 160 mm Hg. Which action should the nurse take for this patient? 1 Administer magnesium sulfate intravenously. 2 Obtain a prescription for antihypertensive medications. 3 Restrict intravenous and oral fluids to 125 mL/hr. 4 Monitor fetal heart rate (FHR) and uterine contractions (UCs).

1, 2, 3 The spotting of blood with the cervical os closed and mild uterine cramping in the first trimester indicates a threatened miscarriage. Therefore the nurse needs to assess progesterone levels, transvaginal ultrasounds, and measurement of hCG to determine whether the fetus is alive and within the uterus. Blood pressure measurements do not help determine the fetal status. KB assay is prescribed to identify fetal-to-maternal bleeding, usually after a trauma.

A pregnant patient in the first trimester reports spotting of blood with the cervical os closed and mild uterine cramping. What does the nurse need to assess? Select all that apply. 1 Progesterone levels 2 Transvaginal ultrasounds 3 Human chorionic gonadotropin (hCG) measurement 4 Blood pressure 5 Kleihauer-Betke (KB) test reports

2 The nurse administers warmed crystalloid solutions for massive fluid resuscitation in the patient who has profound hypovolemia after a trauma. The nurse needs to assess the airway, breathing, and pulse in a patient after a convulsion so that prompt actions can be taken to stabilize the patient. The nurse administers calcium gluconate as an antidote to a patient who has magnesium toxicity. The nurse may administer magnesium sulfate for the treatment of eclamptic seizures in a patient with preeclampsia.

A pregnant patient is at risk for cardiac arrest as a result of profound hypovolemia after a trauma. Which action does the nurse take? The nurse: 1 Assesses airway, breathing, and pulse rate. 2 Administers warmed crystalloid solutions. 3 Administers calcium gluconate intravenously. 4 Obtains a prescription for magnesium sulfate.

1 Abdominal pain in the right lower quadrant, accompanied by nausea and vomiting, indicates appendicitis in a pregnant patient. Cholelithiasis is characterized by right upper quadrant pain. Placenta previa is a condition wherein the placenta is implanted in the lower uterine segment covering the cervix, which causes bleeding when the cervix dilates. Uterine rupture is seen in a pregnant patient as a result of trauma, which may cause fetal death.

A pregnant patient reports abdominal pain in the right lower quadrant, along with nausea and vomiting. The patient's urinalysis report shows an absence of any urinary tract infection in the patient. A chest x-ray also rules out lower-lobe pneumonia. Which condition does the nurse suspect in the patient? 1Appendicitis 2Cholelithiasis 3Placenta previa 4Uterine rupture

2, 3, 5 The nurse instructs the pregnant patient to be alert for abdominal pain, vaginal bleeding, and uterine tenderness as these indicates placental abruption. Weight loss indicates fluid and electrolyte loss and not placental abruption. Shortness of breath indicates inadequate oxygen, which is usually seen in a patient who is having cardiac arrest.

A pregnant patient with chronic hypertension is at risk for placental abruption. Which symptoms of abruption does the nurse instruct the patient to be alert for? Select all that apply. 1 Weight loss 2 Abdominal pain 3 Vaginal bleeding 4 Shortness of breath 5 Uterine tenderness

2, 3, 4 The nurse adds a local anesthetic to the solution to reduce pain that is caused by the injection. The Z-track technique is used to inject the drug so that the drug is injected in the intramuscular (IM) tissue safely. The nurse gently massages the site after administering the injection to reduce pain. The nurse administers two separate injections of 5 g in each buttock. Magnesium sulfate injections are administered in the IM layer and not the subcutaneous layer.

A pregnant patient with severe preeclampsia who is being transported to a tertiary care center needs to be administered magnesium sulfate injection for seizure activity. What actions does the nurse take when administering the drug? Select all that apply. 1 A 10-g dose is administered in the buttock. 2 A local anesthetic is added to the solution. 3 The Z-track technique is used to inject the drug. 4 The injection site is massaged after the injection. 5 The subcutaneous route is used to inject the drug.

2, 3, 5 The diet for hyperemesis includes: (1) Avoid an empty stomach. Eat frequently, at least every 2 to 3 hours. Separate liquids from solids and alternate every 2 to 3 hours. (2) Eat a high-protein snack at bedtime. (3) Eat dry, bland, low-fat, and high-protein foods. Cold foods may be better tolerated than those served at a warm temperature. (4) In general eat what sounds good to you rather than trying to balance your meals. (5) Follow the salty and sweet approach; even so-called junk foods are okay. (6) Eat protein after sweets. (7) Dairy products may stay down more easily than other foods. (8) If you vomit even when your stomach is empty, try sucking on a Popsicle. (9) Try ginger tea. Peel and finely dice a knuckle-sized piece of ginger and place it in a mug of boiling water. Steep for 5 to 8 minutes and add brown sugar to taste. (10) Try warm ginger ale (with sugar, not artificial sweetener) or water with a slice of lemon. (11) Drink liquids from a cup with a lid.

A pregnant woman presents to the emergency department complaining of persistent nausea and vomiting. She is diagnosed with hyperemesis gravidarum. The nurse should include which information when teaching about diet for hyperemesis? Select all that apply. 1 Eat three larger meals a day. 2 Eat a high-protein snack at bedtime. 3 Ice cream may stay down better than other foods. 4 Avoid ginger tea or sweet drinks. 5 Eat what sounds good to you even if your meals are not well-balanced.

1 Bleeding is the most dangerous problem, which impacts the mother's well-being as well as that of her fetus. The decreasing blood volume would cause increases in pulse and respirations and a decrease in blood pressure. The fetus often responds to decreased oxygenation as a result of bleeding, causing a decrease in perfusion. This causes the fetus' heart rate to increase above the normal range of 120--160 beats per minute. The other options have measurements that are in the "normal" range and would not reflect a deterioration of the patient's physical status. Test-Taking Tip: Do not worry if you select the same numbered answer repeatedly, because there usually is no pattern to the answers.

A woman at 37 weeks of gestation is admitted with a placental abruption after a motor vehicle accident. Which assessment data are most indicative of her condition worsening? 1 Pulse (P) 112, respiration (R) 32, blood pressure (BP) 108/60; fetal heart rate (FHR) 166--178 2 P 98, R 22, BP 110/74; FHR 150--162 3 P 88, R 20, BP 114/70; FHR 140--158 4 P 80, R 18, BP 120/78; FHR 138--150

1 Hemorrhage is the most immediate risk because the lower uterine segment has limited ability to contract to reduce blood loss. Infection is a risk because of the location of the placental attachment site; however, it is not a priority concern at this time. Placenta previa poses no greater risk for urinary retention than with a normally implanted placenta. There is no greater risk for thrombophlebitis than with a normally implanted placenta.

A woman diagnosed with marginal placenta previa gave birth vaginally 15 minutes ago. At the present time, she is at the greatest risk for: 1 hemorrhage. 2 infection. 3 urinary retention. 4 thrombophlebitis.

1 Hydralazine is an antihypertensive commonly used to treat hypertension in severe preeclampsia. An additional bolus of magnesium sulfate may be ordered for increasing signs of central nervous system irritability related to severe preeclampsia (e.g., clonus) or if eclampsia develops. Diazepam sometimes is used to stop or shorten eclamptic seizures. Calcium gluconate is used as the antidote for magnesium sulfate toxicity. The patient is not currently displaying any signs or symptoms of magnesium toxicity.

A woman with severe preeclampsia has been receiving magnesium sulfate by intravenous infusion for 8 hours. The nurse assesses the woman and documents the following findings: temperature 37.1° C, pulse rate 96 beats/min, respiratory rate 24 breaths/min, blood pressure 155/112 mm Hg, 3+ deep tendon reflexes, and no ankle clonus. The nurse calls the physician, anticipating an order for: 1hydralazine. 2magnesium sulfate bolus. 3diazepam.

2 Magnesium sulfate is a central nervous system (CNS) depressant given primarily to prevent seizures . A temporary decrease in blood pressure can occur; however, this is not the purpose of administering this medication. Hypotonia is a sign of an excessive serum level of magnesium. It is critical that calcium gluconate be on hand to counteract the depressant effects of magnesium toxicity. Diuresis is not an expected outcome of magnesium sulfate administration.

A woman with severe preeclampsia is being treated with an intravenous infusion of magnesium sulfate. This treatment is considered successful if: 1 blood pressure is reduced to prepregnant baseline. 2 seizures do not occur. 3 deep tendon reflexes become hypotonic. 4 diuresis reduces fluid retention

What symptom described by a woman is characteristic of premenstrual syndrome (PMS)? A) "I feel irritable and moody a week before my period is supposed to start." B) "I have lower abdominal pain beginning the third day of my menstrual period." C) "I have nausea and headaches after my period starts, and they last 2 to 3 days." D) "I have abdominal bloating and breast pain after a couple days of my period."

A) "I feel irritable and moody a week before my period is supposed to start." PMS is a cluster of physical, psychologic, and behavioral symptoms that begin in the luteal phase of the menstrual cycle and resolve within a couple of days of the onset of menses. PMS begins in the luteal phase and resolves as menses occurs. It does not start after menses has begun. This complaint is associated with PMS. However, the timing reflected in this statement is inaccurate. PMS begins in the luteal phase and resolves as menses occurs. It does not start after menses has begun. Abdominal bloating and breast pain are likely to occur a few days prior to menses, not after it has begun.

The CDC-recommended medication for the treatment of chlamydia would be: A) doxycycline. B) podofilox. C) acyclovir. D) penicillin.

A) doxycycline. Doxycycline is effective for treating chlamydia, but it should be avoided if the woman is pregnant. Podofilox is a recommended treatment for nonpregnant women diagnosed with human papilloma virus infection. Acyclovir is recommended for genital herpes simplex virus infection. Penicillin is not a CDC-recommended medication for chlamydia; it is the preferred medication for syphilis.

With regard to dysfunctional uterine bleeding (DUB), the nurse should be aware that: A) it is most commonly caused by anovulation. B) it most often occurs in middle age. C) the diagnosis of DUB should be the first considered for abnormal menstrual bleeding. D) the most effective medical treatment involves steroids.

A) it is most commonly caused by anovulation. Anovulation may occur because of hypothalamic dysfunction or polycystic ovary syndrome. DUB most often occurs when the menstrual cycle is being established or when it draws to a close at menopause. A diagnosis of DUB is made only after all other causes of abnormal menstrual bleeding have been ruled out. The most effective medical treatment is oral or intravenous estrogen.

34. Signs and symptoms that a woman should report immediately to her health care provider include (Select all that apply): a. Vaginal bleeding. b. Rupture of membranes. c. Heartburn accompanied by severe headache. d. Decreased libido. e. Urinary frequency.

A, B, C

46. Examples of sexual risk behaviors associated with exposure to a sexually transmitted infection (STI) include (Select all that apply): a. Fellatio. b. Unprotected anal intercourse. c. Multiple sex partners. d. Dry kissing. e. Abstinence.

A, B, C

26. Intrauterine growth restriction (IUGR) is associated with numerous pregnancy-related risk factors (Select all that apply). a. Poor nutrition b. Maternal collagen disease c. Gestational hypertension d. Premature rupture of membranes e. Smoking

A, B, C, E

27. Transvaginal ultrasonography is often performed during the first trimester. While preparing your 6-week gestation patient for this procedure, she expresses concerns over the necessity for this test. The nurse should explain that this diagnostic test may be indicated for a number of situations (Select all that apply). a. Multifetal gestation b. Obesity c. Fetal abnormalities d. Amniotic fluid volume e. Ectopic pregnancy

A, B, C, E

38. Many pregnant teens wait until the second or third trimester to seek prenatal care. The nurse should understand that the reasons behind this delay include: a. Lack of realization that they are pregnant. b. Uncertainty as to where to go for care. c. Continuing to deny the pregnancy. d. A desire to gain control over their situation. e. Wanting to hide the pregnancy as long as possible.

A, B, C, E

25. Which interventions would help alleviate the problems associated with access to health care for maternity patients (Select all that apply)? a. Provide transportation to prenatal visits. b. Provide childcare so that a pregnant woman may keep prenatal visits. c. Mandate that physicians make house calls. d. Provide low-cost or no-cost health care insurance. e. Provide job training.

A, B, D

45. There is little consensus on the management of premenstrual dysphoric disorder (PMDD). However, nurses can advise women on several self-help modalities that often improve symptoms. The nurse knows that health teaching has been effective when the client reports that she has adopted a number of lifestyle changes, including (Select all that apply): a. Regular exercise. b. Improved nutrition. c. A daily glass of wine. d. Smoking cessation. e. Oil of evening primrose.

A, B, D, E

35. A woman has just moved to the United States from Mexico. She is 3 months pregnant and has arrived for her first prenatal visit. During her assessment interview, you discover that she has not had any immunizations. Which immunizations should she receive at this point in her pregnancy (Select all that apply)? a. Tetanus b. Diphtheria c. Chickenpox d. Rubella e. Hepatitis B

A, B, E

47. The exact cause of breast cancer remains undetermined. Researchers have found that there are many common risk factors that increase a woman's chance of developing a malignancy. It is essential for the nurse who provides care to women of any age to be aware of which of the following risk factors (Select all that apply)? a. Family history b. Late menarche c. Early menopause d. Race e. Nulliparity or first pregnancy after age 30

A, D, E

A key finding from the Human Genome Project is: A. Approximately 20,000 to 25,000 genes make up the genome. B. All human beings are 80.99% identical at the DNA level. C. Human genes produce only one protein per gene; other mammals produce three proteins per gene. D. Single-gene testing will become a standardized test for all pregnant women in the future.

A. Approximately 20,000 to 25,000 genes make up the genome. Approximately 20,500 genes make up the human genome; this is only twice as many as make up the genomes of roundworms and flies. Human beings are 99.9% identical at the DNA level. Most human genes produce at least three proteins. Single-gene testing (e.g., alpha-fetoprotein) is already standardized for prenatal care

The nurse should teach a pregnant woman that which substances are teratogens? (Select all that apply) A. Cigarette smoke B. Isotretinoin (Retin A) C. Vitamin C D. Salicylic acid E. Rubella

A. Cigarette smoke B. Isotretinoin (Retin A) E. Rubella Vitamin C and salicylic acid are not known teratogens.

Which opiate causes euphoria, relaxation, drowsiness, and detachment from reality and has possible effects on the pregnancy, including preeclampsia, intrauterine growth restriction, and premature rupture of membranes? A. Heroin B. Alcohol C. Phencyclidine palmitate (PCP) D. Cocaine

A. Heroin The opiates include opium, heroin, meperidine, morphine, codeine, and methadone. The signs and symptoms of heroin use are euphoria, relaxation, relief from pain, detachment from reality, impaired judgment, drowsiness, constricted pupils, nausea, constipation, slurred speech, and respiratory depression. Possible effects on pregnancy include preeclampsia, intrauterine growth restriction, miscarriage, premature rupture of membranes, infections, breech presentation, and preterm labor. Alcohol is not an opiate. PCP is not an opiate. Cocaine is not an opiate.

The nurse is reviewing lab values to determine Rh incompatibility between mother and fetus. The nurse should assess which specific lab result? A. Indirect Coombs test B. Hemoglobin level C. hCG level D. Maternal serum alpha-fetoprotein (MSAFP)

A. Indirect Coombs test The indirect Coombs test is a screening tool for Rh incompatibility. If the maternal titer for Rh antibodies is greater than 1:8, amniocentesis for determination of bilirubin in amniotic fluid is indicated to establish the severity of fetal hemolytic anemia. Hemoglobin reveals the oxygen carrying capacity of the blood. hCG is the hormone of pregnancy. Maternal serum alpha-fetoprotein (MSAFP) levels are used as a screening tool for NTDs in pregnancy

A maternity nurse should be aware of which fact about the amniotic fluid? A. It serves as a source of oral fluid and as a repository for waste from the fetus. B. The volume remains about the same throughout the term of a healthy pregnancy. C. A volume of less than 300 mL is associated with gastrointestinal malformations. D. A volume of more than 2 L is associated with fetal renal abnormalities.

A. It serves as a source of oral fluid and as a repository for waste from the fetus. Amniotic fluid also cushions the fetus and helps maintain a constant body temperature. The volume of amniotic fluid changes constantly. Too little amniotic fluid (oligohydramnios) is associated with renal abnormalities. Too much amniotic fluid (hydramnios) is associated with gastrointestinal and other abnormalities.

The nurse is caring for a woman with mitral stenosis who is in the active stage. Which action should the nurse take to promote cardiac function? A. Maintain the woman in a side-lying position with the head and shoulders elevated to facilitate hemodynamics B. Prepare the woman for delivery by cesarean section since this is the recommended delivery method to sustain hemodynamics C. Encourage the woman to avoid the use of narcotics or epidural regional analgesia since this alters cardiac function D. Promote the use of the Valsalva maneuver during pushing in the second stage to improve diastolic ventricular filling

A. Maintain the woman in a side-lying position with the head and shoulders elevated to facilitate hemodynamics The side-lying position with the head and shoulders elevated helps to facilitate hemodynamics during labor. A vaginal delivery is the preferred method of delivery for a woman with cardiac disease as it sustains hemodynamics better than a cesarean section. The use of supportive care, medication, and narcotics or epidural regional analgesia is not contraindicated with a woman with heart disease. The use of the Valsalva maneuver during pushing in the second stage should be avoided because it reduces diastolic ventricular filling and obstructs left ventricular outflow.

A nurse is caring for a woman with mitral stenosis who is in the active stage. Which action should the nurse take to promote cardiac function? A. Maintain the woman in a side-lying position with the head and shoulders elevated to facilitate hemodynamics. B. Prepare the woman for delivery by cesarean section because this is the recommended delivery method to sustain hemodynamics. C. Encourage the woman to avoid the use of narcotics or epidural regional analgesia because this alters cardiac function. D. Promote the use of the Valsalva maneuver during pushing in the second stage to improve diastolic ventricular filling.

A. Maintain the woman in a side-lying position with the head and shoulders elevated to facilitate hemodynamics. The side-lying position with the head and shoulders elevated helps facilitate hemodynamics during labor. A vaginal delivery is the preferred method for a woman with cardiac disease because it sustains hemodynamics better than a cesarean section. The use of supportive care, medication, and narcotics or epidural regional analgesia is not contraindicated with a woman with heart disease. Epidural anesthesia for labor is preferred. (Easterling and Stout, 2012). Using the Valsalva maneuver during pushing in the second stage should be avoided because it reduces diastolic ventricular filling and obstructs left ventricular outflow.

A pregnant woman is being examined by the nurse in the outpatient obstetric clinic. The nurse suspects systemic lupus erythematosus (SLE) after revealing which symptoms? (Select all that apply.) A. Muscle aches B. Hyperactivity C. Weight changes D. Fever E. Hypotension

A. Muscle aches C. Weight changes D. Fever Fatigue, rather than hyperactivity is a common sign of SLE. Hypotension is not a characteristic sign of SLE. Common symptoms, including myalgias, fatigue, weight change, and fevers, occur in nearly all women with SLE at some time during the course of the disease. Although a diagnosis of SLE is suspected based on clinical signs and symptoms, it is confirmed by laboratory testing that demonstrates the presence of circulating autoantibodies. As is the case with other autoimmune diseases, SLE is characterized by a series of exacerbations (flares) and remissions (Chin and Branch, 2012).

Most of the genetic tests now offered in clinical practice are tests for: A. Single-gene disorders. B. Carrier screening. C. Predictive values. D. Predispositional testing.

A. Single-gene disorders. Most tests now offered are tests for single-gene disorders in clients with clinical symptoms or who have a family history of a genetic disease. Carrier screening is used to identify individuals who have a gene mutation for a genetic condition but do not display symptoms. Predictive testing is used only to clarify the genetic status of asymptomatic family members. Predispositional testing differs from the other types of genetic screening in that a positive result does not indicate a 100% chance of developing the condition.

Thalassemia is a relatively common anemia in which: A. an insufficient amount of hemoglobin is produced to fill the red blood cells (RBCs). B. RBCs have a normal life span but are sickled in shape. C. folate deficiency occurs. D. there are inadequate levels of vitamin B12.

A. an insufficient amount of hemoglobin is produced to fill the red blood cells (RBCs). Thalassemia is a hereditary disorder that involves the abnormal synthesis of the á or â chains of hemoglobin. An insufficient amount of hemoglobin is produced to fill the RBCs. This is the underlying description for sickle cell anemia. Folate deficiency is the most common cause of megaloblastic anemias during pregnancy. B12 deficiency must also be considered if the pregnant woman presents with anemia.

Hypothyroidism occurs in 2 to 3 pregnancies per 1000. Pregnant women with untreated hypothyroidism are at risk for: (Select all that apply.) A. miscarriage. B. macrosomia. C. gestational hypertension. D. placental abruption. E. stillbirth.

A. miscarriage. C. gestational hypertension. D. placental abruption. E. stillbirth. Hypothyroidism is often associated with both infertility and an increased risk of miscarriage. Infants born to mothers with hypothyroidism are more likely to be of low birth weight or preterm. These outcomes can be improved with early diagnosis and treatment. Pregnant women with hypothyroidism are more likely to experience both preeclampsia and gestational hypertension. Placental abruption and stillbirth are risks associated with hypothyroidism. Placental abruption and stillbirth are risks associated with hypothyroidism.

Prior to the patient undergoing amniocentesis, the most appropriate nursing intervention is to: A: administer RhoD immunoglobulin. B. send the patient for a computed tomography (CT) scan before the procedure. C. assure the mother that short-term radiation exposure is not harmful to the fetus. D. administer anticoagulant.

A: administer RhoD immunoglobulin. Because of the possibility of fetomaternal hemorrhage, administering RhoD immunoglobulin to the woman who is Rh negative is standard practice after an amniocentesis. Anticoagulants are not administered before amniocentesis as this would increase the risk of bleeding when the needle is inserted transabdominally. A CT is not required before amniocentesis, because the procedure is ultrasound guided. The mother is not exposed to radiation during amniocentesis.

6. In vitro fertilization-embryo transfer (IVF-ET) is a common approach for women with blocked fallopian tubes or unexplained infertility and for men with very low sperm counts. A husband and wife have arrived for their preprocedural interview. The husband asks the nurse to explain what the procedure entails. The nurse's most appropriate response is: a. "IVF is a type of assisted reproductive therapy that involves collecting eggs from your wife's ovaries, fertilizing them in the lab with your sperm, and transferring the embryo to her uterus." b. "A donor embryo will be transferred into your wife's uterus." c. "Donor sperm will be used to inseminate your wife." d. "Don't worry about the technical stuff; that's what we are here for."

ANS: A A woman's eggs are collected from her ovaries, fertilized in the laboratory with sperm, and transferred to her uterus after normal embryonic development has occurred. The statement, "A donor embryo will be transferred into your wife's uterus" describes therapeutic donor insemination. "Donor sperm will be used to inseminate your wife" describes the procedure for a donor embryo. "Don't worry about the technical stuff; that's what we are here for" discredits the client's need for teaching and is not the most appropriate response.

11. A woman has chosen the calendar method of conception control. During the assessment process it is most important that the nurse: a. Obtain a history of menstrual cycle lengths for the past 6 to 12 months. b. Determine the client's weight gain and loss pattern for the previous year. c. Examine skin pigmentation and hair texture for hormonal changes. d. Explore the client's previous experiences with conception control.

ANS: A The calendar method of conception control is based on the number of days in each cycle, counting from the first day of menses. The fertile period is determined after the lengths of menstrual cycles have been accurately recorded for 6 months. Weight gain or loss may be partly related to hormonal fluctuations, but it has no bearing on use of the calendar method. Integumentary changes may be related to hormonal changes, but they are not indicators for use of the calendar method. Exploring previous experiences with conception control may demonstrate client understanding and compliancy, but it is not the most important aspect to assess for discussion of the calendar method.

20. A woman is 16 weeks pregnant and has elected to terminate her pregnancy. The nurse knows that the most common technique used for medical termination of a pregnancy in the second trimester is: a. Dilation and evacuation (D&E). b. Instillation of hypertonic saline into the uterine cavity. c. Intravenous administration of Pitocin. d. Vacuum aspiration.

ANS: A The most common technique for medical termination of a pregnancy in the second trimester is D&E. It is usually performed between 13 and 16 weeks. Hypertonic solutions injected directly into the uterus account for less than 1% of all abortions because other methods are safer and easier to use. Intravenous administration of Pitocin is used to induce labor in a woman with a third-trimester fetal demise. Vacuum aspiration is used for abortions in the first trimester.

25. Which contraceptive method has a failure rate of less than 25%? a. Standard days c. Postovulation b. Periodic abstinence d. Coitus interruptus

ANS: A The standard days variation on the calendar method has a failure rate of 12%. The periodic abstinence method has a failure rate of 25% or higher. The postovulation method has a failure rate of 25% or higher. The coitus interruptus method has a failure rate of 27% or higher.

7. Nurses should be aware that infertility: a. Is perceived differently by women and men. b. Has a relatively stable prevalence among the overall population and throughout a women's potential reproductive years. c. Is more likely the result of a physical flaw in the woman than in her male partner. d. Is the same thing as sterility.

ANS: A Women tend to be more stressed about infertility tests and to place more importance on having children. The prevalence of infertility is stable among the overall population, but it increases with a woman's age, especially over age 40. Of cases with an identifiable cause, about 40% are related to female factors, 40% to male factors, and 20% to both partners. Sterility is the inability to conceive. Infertility is a state of requiring a prolonged time to conceive, or subfertility.

18. A woman was treated recently for toxic shock syndrome (TSS). She has intercourse occasionally and uses over-the-counter protection. On the basis of her history, what contraceptive method should she and her partner avoid? a. Cervical cap c. Vaginal film b. Condom d. Vaginal sheath

ANS: A Women with a history of TSS should not use a cervical cap. Condoms, vaginal films, and vaginal sheaths are not contraindicated for a woman with a history of TSS.

31. You (the nurse) are reviewing the educational packet provided to a client about tubal ligation. What is an important fact you should point out? Choose all that apply. a. "It is highly unlikely that you will become pregnant after the procedure." b. "This is an effective form of 100% permanent sterilization. You won't be able to get pregnant." c. "Sterilization offers some form of protection against sexually transmitted infections (STIs)." d. "Sterilization offers no protection against STIs." e. "Your menstrual cycle will greatly increase after your sterilization."

ANS: A, D A woman is unlikely to become pregnant after tubal ligation, although it is not 100% effective. It does not offer any protection against STIs. Sterilization offers no protection against STIs. Typically the menstrual cycle remains the same after a tubal ligation.

14. A male client asks the nurse why it is better to purchase condoms that are not lubricated with nonoxynol-9 (a common spermicide). The nurse's most appropriate response is: a. "The lubricant prevents vaginal irritation." b. "Nonoxynol-9 does not provide protection against sexually transmitted infections, as originally thought; it has also been linked to an increase in the transmission of human immunodeficiency virus and can cause genital lesions." c. "The additional lubrication improves sex." d. "Nonoxynol-9 improves penile sensitivity."

ANS: B "Nonoxynol-9 does not provide protection against sexually transmitted infections, as originally thought; it has also been linked to an increase in the transmission of human immunodeficiency virus and can cause genital lesions" is a true statement. Nonoxynol-9 may cause vaginal irritation, has no effect on the quality of sexual activity, and has no effect on penile sensitivity.

26. Which contraceptive method best protects against sexually transmitted infections (STIs) and human immunodeficiency virus (HIV)? a. Periodic abstinence b. Barrier methods c. Hormonal methods d. They all offer about the same protection.

ANS: B Barrier methods such as condoms best protect against STIs and HIV. Periodic abstinence and hormonal methods ("the pill") offer no protection against STIs or HIV.

15. A woman who has a seizure disorder and takes barbiturates and phenytoin sodium daily asks the nurse about the pill as a contraceptive choice. The nurse's most appropriate response would be: a. "This is a highly effective method, but it has some side effects." b. "Your current medications will reduce the effectiveness of the pill." c. "The pill will reduce the effectiveness of your seizure medication." d. "This is a good choice for a woman of your age and personal history."

ANS: B Because the liver metabolizes oral contraceptives, their effectiveness is reduced when they are taken simultaneously with anticonvulsants. "Your current medications will reduce the effectiveness of the pill" is a true statement, but it is not the most appropriate response. The anticonvulsant will reduce the effectiveness of the pill, not the other way around. "This is a good choice for a woman of your age and personal history" does not teach the client that the effectiveness of the pill may be reduced because of her anticonvulsant therapy.

19. An unmarried young woman describes her sex life as "active" and involving "many" partners. She wants a contraceptive method that is reliable and does not interfere with sex. She requests an intrauterine device (IUD). The nurse's most appropriate response is: a. "The IUD does not interfere with sex." b. "The risk of pelvic inflammatory disease (PID) will be higher for you." c. "The IUD will protect you from sexually transmitted infections (STIs)." d. "Pregnancy rates are high with the IUDs."

ANS: B Disadvantages of IUDs include an increased risk of PID in the first 20 days after insertion and the risks of bacterial vaginosis and uterine perforation. The IUD offers no protection against STIs or the human immunodeficiency virus. Because this woman has multiple sex partners, she is at higher risk of developing an STI. The IUD does not protect against infection, as does a barrier method. Although "The IUD does not interfere with sex" may be correct, it is not the most appropriate response. The IUD offers no protection from STIs. The typical failure rate of the IUD ranges from 0.8% to 2%.

1. Which test used to diagnose the basis of infertility is done during the luteal or secretory phase of the menstrual cycle? a. Hysterosalpingogram b. Endometrial biopsy c. Laparoscopy d. Follicle-stimulating hormone (FSH) level

ANS: B Endometrial biopsy is scheduled after ovulation, during the luteal phase of the menstrual cycle. A hysterosalpingogram is scheduled 2 to 5 days after menstruation to avoid flushing potentially fertilized ovum out through a uterine tube into the peritoneal cavity. Laparoscopy usually is scheduled early in the menstrual cycle. Hormone analysis is performed to assess endocrine function of the hypothalamic-pituitary-ovarian axis when menstrual cycles are absent or irregular.

13. A married couple is discussing alternatives for pregnancy prevention and has asked about fertility awareness methods (FAMs). The nurse's most appropriate reply is: a. "They're not very effective, and it's very likely you'll get pregnant." b. "They can be effective for many couples, but they require motivation." c. "These methods have a few advantages and several health risks." d. "You would be much safer going on the pill and not having to worry."

ANS: B FAMs are effective with proper vigilance about ovulatory changes in the body and adherence to coitus intervals. They are effective if used correctly by a woman with a regular menstrual cycle. The typical failure rate for all FAMs is 25% during the first year of use. FAMs have no associated health risks. The use of birth control has associated health risks. In addition, taking a pill daily requires compliance on the client's part.

8. With regard to the assessment of female, male, and couple infertility, nurses should be aware that: a. The couple's religious, cultural, and ethnic backgrounds provide emotional clutter that does not affect the clinical scientific diagnosis. b. The investigation takes 3 to 4 months and a lot of money. c. The woman is assessed first; if she is not the problem, the male partner is analyzed. d. Semen analysis is for men; the postcoital test is for women.

ANS: B Fertility assessment and diagnosis take time, money, and commitment from the couple. Religious, cultural, and ethnic-bred attitudes about fertility and related issues always have an impact on diagnosis and assessment. Both partners are assessed systematically and simultaneously, as individuals and as a couple. Semen analysis is for men, but the postcoital test is for the couple.

5. A woman inquires about herbal alternative methods for improving fertility. Which statement by the nurse is the most appropriate when instructing the client in which herbal preparations to avoid while trying to conceive? a. "You should avoid nettle leaf, dong quai, and vitamin E while you are trying to get pregnant." b. "You may want to avoid licorice root, lavender, fennel, sage, and thyme while you are trying to conceive." c. "You should not take anything with vitamin E, calcium, or magnesium. They will make you infertile." d. "Herbs have no bearing on fertility."

ANS: B Herbs that a woman should avoid while trying to conceive include licorice root, yarrow, wormwood, ephedra, fennel, golden seal, lavender, juniper, flaxseed, pennyroyal, passionflower, wild cherry, cascara, sage, thyme, and periwinkle. Nettle leaf, dong quai, and vitamin E are all remedies that promote fertility. Vitamin E, calcium, and magnesium may promote fertility and conception. Although most herbal remedies have not been proven clinically to promote fertility, women should avoid the following herbs while trying to conceive: licorice root, yarrow, wormwood, ephedra, fennel, golden seal, lavender, juniper, flaxseed, pennyroyal, passionflower, wild cherry, cascara, sage, thyme, and periwinkle.

28. With regard to the use of intrauterine devices (IUDs), nurses should be aware that: a. Return to fertility can take several weeks after the device is removed. b. IUDs containing copper can provide an emergency contraception option if inserted within a few days of unprotected intercourse. c. IUDs offer the same protection against sexually transmitted infections as the diaphragm. d. Consent forms are not needed for IUD insertion.

ANS: B The woman has up to 8 days to insert the IUD after unprotected sex. Return to fertility is immediate after removal of the IUD. IUDs offer no protection for sexually transmitted infections. A consent form is required for insertion, as is a negative pregnancy test.

30. Nurses, certified nurse-midwives, and other advanced practice nurses have the knowledge and expertise to assist women in making informed choices regarding contraception. A multidisciplinary approach should ensure that the woman's social, cultural, and interpersonal needs are met. Which action should the nurse take first when meeting with a new client to discuss contraception? a. Obtain data about the frequency of coitus. b. Determine the woman's level of knowledge about contraception and commitment to any particular method. c. Assess the woman's willingness to touch her genitals and cervical mucus. d. Evaluate the woman's contraceptive life plan.

ANS: B This is the primary step of this nursing assessment and necessary before completing the process and moving on to a nursing diagnosis. Once the client's level of knowledge is determined, the nurse can interact with the woman to compare options, reliability, cost, comfort level, protection from sexually transmitted infections, and a partner's willingness to participate. Although important, obtaining data about the frequency of coitus is not the first action that the nurse should undertake when completing an assessment. Data should include not only the frequency of coitus but also the number of sexual partners, level of contraceptive involvement, and partner's objections. Assessing the woman's willingness to touch herself is a key factor for the nurse to discuss should the client express interest in using one of the fertility awareness methods of contraception. The nurse must be aware of the client's plan regarding whether she is attempting to prevent conception, delay conception, or conceive.

16. Injectable progestins (DMPA, Depo-Provera) are a good contraceptive choice for women who: a. Want menstrual regularity and predictability. b. Have a history of thrombotic problems or breast cancer. c. Have difficulty remembering to take oral contraceptives daily. d. Are homeless or mobile and rarely receive health care.

ANS: C Advantages of DMPA include a contraceptive effectiveness comparable to that of combined oral contraceptives with the requirement of only four injections a year. Disadvantages of injectable progestins are prolonged amenorrhea and uterine bleeding. Use of injectable progestin carries an increased risk of venous thrombosis and thromboembolism. To be effective, DMPA injections must be administered every 11 to 13 weeks. Access to health care is necessary to prevent pregnancy or potential complications.

24. While instructing a couple regarding birth control, the nurse should be aware that the method called natural family planning: a. Is the same as coitus interruptus, or "pulling out." b. Uses the calendar method to align the woman's cycle with the natural phases of the moon. c. Is the only contraceptive practice acceptable to the Roman Catholic church. d. Relies on barrier methods during fertility phases.

ANS: C Natural family planning is the only contraceptive practice acceptable to the Roman Catholic church. "Pulling out" is not the same as periodic abstinence, another name for natural family planning. The phases of the moon are not part of the calendar method or any method. Natural family planning is another name for periodic abstinence, which is the accepted way to pass safely through the fertility phases without relying on chemical or physical barriers.

3. A couple comes in for an infertility workup, having attempted to get pregnant for 2 years. The woman, 37, has always had irregular menstrual cycles but is otherwise healthy. The man has fathered two children from a previous marriage and had a vasectomy reversal 2 years ago. The man has had two normal semen analyses, but the sperm seem to be clumped together. What additional test is needed? a. Testicular biopsy b. Antisperm antibodies c. Follicle-stimulating hormone (FSH) level d. Examination for testicular infection

ANS: C The woman has irregular menstrual cycles. The scenario does not indicate that she has had any testing related to this irregularity. Hormone analysis is performed to assess endocrine function of the hypothalamic-pituitary-ovarian axis when menstrual cycles are absent or irregular. Determination of blood levels of prolactin, FSH, luteinizing hormone (LH), estradiol, progesterone, and thyroid hormones may be necessary to diagnose the cause of irregular menstrual cycles. A testicular biopsy would be indicated only in cases of azoospermia (no sperm cells) or severe oligospermia (low number of sperm cells). Antisperm antibodies are produced by a man against his own sperm. This is unlikely to be the case here, because the husband has already produced children. Examination for testicular infection would be done before semen analysis. Furthermore, infection would affect spermatogenesis.

21. A woman will be taking oral contraceptives using a 28-day pack. The nurse should advise this woman to protect against pregnancy by: a. Limiting sexual contact for one cycle after starting the pill. b. Using condoms and foam instead of the pill for as long as she takes an antibiotic. c. Taking one pill at the same time every day. d. Throwing away the pack and using a backup method if she misses two pills during week 1 of her cycle.

ANS: C To maintain adequate hormone levels for contraception and to enhance compliance, clients should take oral contraceptives at the same time each day. If contraceptives are to be started at any time other than during normal menses or within 3 weeks after birth or abortion, another method of contraception should be used through the first week to prevent the risk of pregnancy. Taken exactly as directed, oral contraceptives prevent ovulation, and pregnancy cannot occur. No strong pharmacokinetic evidence indicates a link between the use of broad-spectrum antibiotics and altered hormone levels in oral contraceptive users. If the client misses two pills during week 1, she should take two pills a day for 2 days, finish the package, and use a backup method the next 7 consecutive days.

22. A woman had unprotected intercourse 36 hours ago and is concerned that she may become pregnant because it is her "fertile" time. She asks the nurse about emergency contraception. The nurse tells her that: a. It is too late; she needed to begin treatment within 24 hours after intercourse. b. Preven, an emergency contraceptive method, is 98% effective at preventing pregnancy. c. An over-the-counter antiemetic can be taken 1 hour before each contraceptive dose to prevent nausea and vomiting. d. The most effective approach is to use a progestin-only preparation.

ANS: C To minimize the side effect of nausea that occurs with high doses of estrogen and progestin, the woman can take an over-the-counter antiemetic 1 hour before each dose. Emergency contraception is used within 72 hours of unprotected intercourse to prevent pregnancy. Postcoital contraceptive use is 74% to 90% effective at preventing pregnancy. Oral emergency contraceptive regimens may include progestin-only and estrogen-progestin pills. Women with contraindications to estrogen use should use progestin-only pills.

4. A couple is trying to cope with an infertility problem. They want to know what they can do to preserve their emotional equilibrium. The nurse's most appropriate response is: a. "Tell your friends and family so they can help you." b. "Talk only to other friends who are infertile because only they can help." c. "Get involved with a support group. I'll give you some names." d. "Start adoption proceedings immediately because it is very difficult to obtain an infant."

ANS: C Venting negative feelings may unburden the couple. A support group may provide a safe haven for the couple to share their experiences and gain insight from others' experiences. Although talking about their feelings may unburden them of negative feelings, infertility can be a major stressor that affects the couple's relationships with family and friends. Limiting their interactions to other infertile couples may be a beginning point for addressing psychosocial needs, but depending on where the other couple is in their own recovery process, this may or may not help them. The statement about adoption proceedings is not supportive of the psychosocial needs of this couple and may be detrimental to their well-being

10. Although remarkable developments have occurred in reproductive medicine, assisted reproductive therapies are associated with a number of legal and ethical issues. Nurses can provide accurate information about the risks and benefits of treatment alternatives so couples can make informed decisions about their choice of treatment. Which issue would not need to be addressed by an infertile couple before treatment? a. Risks of multiple gestation b. Whether or how to disclose the facts of conception to offspring c. Freezing embryos for later use d. Financial ability to cover the cost of treatment

ANS: D Although the method of payment is important, obtaining this information is not the responsibility of the nurse. It is also of note that 14 states have mandated some form of insurance to assist couples with coverage for infertility. Risks of multiple gestation is indeed a risk of treatment of which the couple needs to be aware. To minimize the chance of multiple gestation, generally only three or fewer embryos are transferred. The couple should be informed that there may be a need for multifetal reduction. Nurses can provide anticipatory guidance on this matter. Depending on the therapy chosen, there may be a need for donor oocytes, sperm, embryos, or a surrogate mother. Couples who have excess embryos frozen for later transfer must be fully informed before consenting to the procedure. A decision must be made regarding the disposal of embryos in the event of death or divorce or if the couple no longer wants the embryos at a future time.

23. Which statement is true about the term contraceptive failure rate? a. It refers to the percentage of users expected to have an accidental pregnancy over a 5-year span. b. It refers to the minimum level that must be achieved to receive a government license. c. It increases over time as couples become more careless. d. It varies from couple to couple, depending on the method and the users.

ANS: D Contraceptive effectiveness varies from couple to couple, depending on how well a contraceptive method is used and how well it suits the couple. The contraceptive failure rate measures the likelihood of accidental pregnancy in the first year only. Failure rates decline over time because users gain experience.

29. Which statement is the most complete and accurate description of medical abortions? a. They are performed only for maternal health. b. They can be achieved through surgical procedures or with drugs. c. They are mostly performed in the second trimester. d. They can be either elective or therapeutic.

ANS: D Medical abortions are performed through the use of medications (rather than surgical procedures). They are mostly done in the first trimester, and they can be either elective (the woman's choice) or therapeutic (for reasons of maternal or fetal health).

9. In their role of implementing a plan of care for infertile couples, nurses should: a. Be comfortable with their sexuality and nonjudgmental about others to effectively counsel their clients. b. Know about such nonmedical remedies as diet, exercise, and stress management. c. Be able to direct clients to sources of information about what herbs to take that might help and which ones to avoid. d. Do all of the above plus be knowledgeable about potential drug and surgical remedies.

ANS: D Nurses should be open to and ready to help with a variety of medical and nonmedical approaches.

12. A woman is using the basal body temperature (BBT) method of contraception. She calls the clinic and tells the nurse, "My period is due in a few days, and my temperature has not gone up." The nurse's most appropriate response is: a. "This probably means that you're pregnant." b. "Don't worry; it's probably nothing." c. "Have you been sick this month?" d. "You probably didn't ovulate during this cycle."

ANS: D The absence of a temperature decrease most likely is the result of lack of ovulation. Pregnancy cannot occur without ovulation (which is being measured using the BBT method). A comment such as "Don't worry; it's probably nothing" discredits the client's concerns. Illness would most likely cause an increase in basal body temperature.

17. A woman currently uses a diaphragm and spermicide for contraception. She asks the nurse what the major differences are between the cervical cap and diaphragm. The nurse's most appropriate response is: a. "No spermicide is used with the cervical cap, so it's less messy." b. "The diaphragm can be left in place longer after intercourse." c. "Repeated intercourse with the diaphragm is more convenient." d. "The cervical cap can safely be used for repeated acts of intercourse without adding more spermicide later."

ANS: D The cervical cap can be inserted hours before sexual intercourse without the need for additional spermicide later. No additional spermicide is required for repeated acts of intercourse. Spermicide should be used inside the cap as an additional chemical barrier. The cervical cap should remain in place for 6 hours after the last act of intercourse. Repeated intercourse with the cervical cap is more convenient, because no additional spermicide is needed.

27. With regard to the noncontraceptive medical effects of combined oral contraceptive pills (COCs), nurses should be aware that: a. COCs can cause toxic shock syndrome if the prescription is wrong. b. Hormonal withdrawal bleeding usually is a bit more profuse than in normal menstruation and lasts a week. c. COCs increase the risk of endometrial and ovarian cancer. d. The effectiveness of COCs can be altered by some over-the-counter medications and herbal supplements.

ANS: D The effectiveness of COCs can be altered by some over-the-counter medications and herbal supplements. Toxic shock syndrome can occur in some diaphragm users, but it is not a consequence of taking oral contraceptive pills. Hormonal withdrawal bleeding usually is lighter than in normal menstruation and lasts a couple of days. Oral contraceptive pills offer protection against the risk of endometrial and ovarian cancers.

2. A man smokes two packs of cigarettes a day. He wants to know if smoking is contributing to the difficulty he and his wife are having getting pregnant. The nurse's most appropriate response is: a. "Your sperm count seems to be okay in the first semen analysis." b. "Only marijuana cigarettes affect sperm count." c. "Smoking can give you lung cancer, even though it has no effect on sperm." d. "Smoking can reduce the quality of your sperm."

ANS: D Use of tobacco, alcohol, and marijuana may affect sperm counts. "Your sperm count seems to be okay in the first semen analysis" is inaccurate. Sperm counts vary from day to day and depend on emotional and physical status and sexual activity. Therefore a single analysis may be inconclusive. A minimum of two analyses must be performed several weeks apart to assess male fertility. Use of tobacco, alcohol, and marijuana may affect sperm counts.

1 Once the vomiting has stopped, feedings are started in small amounts at frequent intervals. In the beginning, limited amounts of oral fluids and bland foods such as crackers, toast, or baked chicken are offered. Clear fluids alone do not contain enough calories and contain no protein. Most women are able to take nourishment by mouth after several days of treatment. They should be encouraged to eat small, frequent meals and foods that sound appealing (e.g., nongreasy, dry, sweet, and salty foods). Test-Taking Tip: Many times the correct answer is the longest alternative given, but do not count on it. NCLEX item writers (those who write the questions) are also aware of this and attempt to avoid offering you such "helpful hints."

After being rehydrated in the emergency department, a 24 year-old primipara in her 18th week of pregnancy is at home and is to rest at home for the next two days and take in small but frequent fluids and food as possible. Discharge teaching at the hospital by the nurse has been effective if the patient makes which statement? 1"I'm going to eat five to six small servings per day, which contain such foods and fluids as tea, crackers, or a few bites of baked potato." 2 A strip of bacon and a fried egg will really taste good as long as I eat them slowly." 3"As long as I eat small amounts and allow enough time for digestion, I can eat almost anything, like barbequed chicken or spaghetti." 4"I'm going to stay only on clear fluids for the next 24 hours and then add dairy products like eggs and milk."

3 Full assessment of the patient and her fetus are essential and include vital signs, continual fetal heart rate monitoring, determining the location and severity of pain, whether any vaginal bleeding is dark red or bright red, and the status of the abdomen, which would be expected to be rigid or "board like." Staying with the patient, assuring a patent airway is present, and keeping the patient as calm as possible would be appropriate at the crash site before the arrival of emergency medical services (EMS). The current status of the patient and fetus are thepriority. The health care provider would prescribe the arterial blood gases and other laboratory work after the patient is assessed and stabilized.

At 37 weeks of gestation, the patient is in a severe automobile crash where her abdomen was hit by the steering wheel and her seat belt. What actions would the emergency room nurse expect to perform upon the patient's arrival at the hospital? 1 Stay with the patient, assure a patent airway is present, and keep the patient as calm as possible. 2 Move the patient's skirt to determine if any vaginal bleeding is present, find out who to call, and monitor the level of consciousness. 3 Assess the patient's vital signs, determine location and severity of pain, and establish continual fetal heart rate monitoring. 4 Obtain arterial blood gases, obtain a hemoglobin and hematocrit, and oxygen saturation rate.

With regard to the diagnosis and management of amenorrhea, nurses should be aware that: A) it probably is the result of a hormone deficiency that can be treated with medication. B) it may be caused by stress or excessive exercise or both. C) it likely will require the client to eat less and exercise more. D) it often goes away on its own.

B) it may be caused by stress or excessive exercise or both. Amenorrhea may be the result of a decrease in follicle-stimulating hormone (FSH) and luteinizing hormone (LH). This is usually caused by stress, body fat to lean ratio, and in rare occurrences a pituitary tumor. It cannot be treated by medication. Amenorrhea usually is the result of stress and/or an inappropriate ratio of body fat to lean tissue, possibly as a result of excessive exercise. Management includes counseling and education about the causes and possible lifestyle changes. In most cases a client will need to decrease her amount of exercise and increase her body weight in order to resume menstruation. Management of stress and eating disorders is usually necessary to manage this condition.

When providing care to a young single woman just diagnosed with acute pelvic inflammatory disease, the nurse should: A) point out that inappropriate sexual behavior caused the infection. B) position the woman in a semi-Fowler position. C) explain to the woman that infertility is a likely outcome of this type of infection. D) tell her that antibiotics need to be taken until pelvic pain is relieved.

B) position the woman in a semi-Fowler position. Although sexual behavior may have contributed to the infection, the nurse must discuss these practices in a nonjudgmental manner and provide information about prevention measures. The position of comfort is the semi-Fowler position. In addition, the foot of the bed could be elevated to keep the uterus in a dependent position and reduce discomfort. Until treatment is complete and healing has occurred, the outcome is unknown and should not be suggested. The nurse should emphasize that medication must be continued until follow-up assessment indicates that the infection has been treated successfully.

26. Which of the following statements indicate that the nurse is practicing appropriate family-centered care techniques (Select all that apply)? a. The nurse commands the mother to do as she is told. b. The nurse allows time for the partner to ask questions. c. The nurse allows the mother and father to make choices when possible. d. The nurse informs the family about what is going to happen. e. The nurse tells the patient's sister, who is a nurse, that she cannot be in the room during the delivery.

B, C

A nurse counseling a client with endometriosis understands which statements regarding the management of endometriosis is accurate? (Select all that apply) A) Bone loss from hypoestrogenism is not reversible. B) Side effects from the steroid danazol include masculinizing traits. C) Surgical intervention often is needed for severe or acute symptoms. D) Women without pain and who do not want to become pregnant need no treatment. E) Women with mild pain who may want a future pregnancy may take nonsteroidal antiinflammatory drugs (NSAIDs).

B, C, D B) Side effects from the steroid danazol include masculinizing traits, C) Surgical intervention often is needed for severe or acute symptoms and D) Women without pain and who do not want to become pregnant need no treatment. Bone loss is mostly reversible within 12 to 18 months after the medication is stopped. Such masculinizing traits as hirsutism, a deepening voice, and weight gain occur with danazol but are reversible. Surgical intervention often is needed when symptoms are incapacitating. The type of surgery is influenced by the woman's age and desire to have children. Treatment is not needed for women without pain or the desire to have children. In women with mild pain who may desire a future pregnancy, treatment may be limited to use of NSAIDs during menstruation.

The nurse should include which information when teaching a 15-year-old about genital tract infection prevention? (Select all that apply.) A) Wear nylon undergarments. B) Avoid tight-fitting jeans. C) Use floral scented bath salts. D) Decrease sugar intake. E) Do not douche. F) Limit time spent wearing a wet bathing suit

B, D, E B) Avoid tight-fitting jeans, D) Decrease sugar intake & E) Do not douche. Patient teaching for the prevention of genital tract infections in women includes the following guidelines: • Practice genital hygiene. • Choose underwear or hosiery with a cotton crotch. • Avoid tight-fitting clothing (especially tight jeans). • Select cloth car seat covers instead of vinyl. • Limit the time spent in damp exercise clothes (especially swimsuits, leotards, and tights). • Limit exposure to bath salts or bubble bath. • Avoid colored or scented toilet tissue. • If sensitive, discontinue use of feminine hygiene deodorant sprays. • Use condoms. • Void before and after intercourse. • Decrease dietary sugar. • Drink yeast-active milk and eat yogurt (with lactobacilli). • Do not douche. Incorrect Feedback: Patient teaching for the prevention of genital tract infections in women includes the following guidelines: • Practice genital hygiene. • Choose underwear or hosiery with a cotton crotch. • Avoid tight-fitting clothing (especially tight jeans). • Select cloth car seat covers instead of vinyl. • Limit the time spent in damp exercise clothes (especially swimsuits, leotards, and tights). • Limit exposure to bath salts or bubble bath. • Avoid colored or scented toilet tissue. • If sensitive, discontinue use of feminine hygiene deodorant sprays. • Use condoms. • Void before and after intercourse. • Decrease dietary sugar. • Drink yeast-active milk and eat yogurt (with lactobacilli). • Do not douche.

A woman is 8 months pregnant. She tells the nurse that she knows her baby listens to her, but her husband thinks she is imagining things. Which response by the nurse is most appropriate? A. "Many women imagine what their baby is like." B. "A baby in utero does respond to the mother's voice." C. "You'll need to ask the doctor if the baby can hear yet." D. "Thinking that your baby hears will help you bond with the baby."

B. "A baby in utero does respond to the mother's voice." Although this statement is accurate, it is not the most appropriate response. Fetuses respond to sound by 24 weeks. The fetus can be soothed by the sound of the mother's voice. This statement is not appropriate. The mother should be instructed that her fetus can hear at 24 weeks and can respond to the sound of her voice. The statement is not appropriate. It gives the impression that her baby cannot hear her. It also belittles the mother's interpretation of her fetus's behaviors.

With regard to abnormalities of chromosomes, nurses should be aware that: A. They occur in approximately 10% of newborns. B. Abnormalities of number are the leading cause of pregnancy loss. C. Down syndrome is a result of an abnormal chromosomal structure. D. Unbalanced translocation results in a mild abnormality that the child will outgrow.

B. Abnormalities of number are the leading cause of pregnancy loss. Chromosomal abnormalities occur in less than 1% of newborns. Aneuploidy is an abnormality of number that also is the leading genetic cause of mental retardation. Down syndrome is the most common form of trisomal abnormality, an abnormality of chromosome number (47 chromosomes). Unbalanced translocation is an abnormality of chromosome structure that often has serious clinical effects.

What best describes the pattern of genetic transmission known as autosomal recessive inheritance? A. Disorders in which the abnormal gene for the trait is expressed even when the other member of the pair is normal B. Disorders in which both genes of a pair must be abnormal for the disorder to be expressed C. Disorders in which a single gene controls the particular trait D. Disorders in which the abnormal gene is carried on the X chromosome

B. Disorders in which both genes of a pair must be abnormal for the disorder to be expressed Autosomal dominant inheritance occurs when the abnormal gene for the trait is expressed, even when the other member of the pair is normal, such as Huntington disease or Marfan syndrome. An autosomal recessive inheritance disorder occurs when both genes of the pair are abnormal, such as phenylketonuria or sickle cell anemia. Disorders in which a single gene controls the particular trait describe the unifactorial inheritance. X-linked recessive inheritance occurs when the abnormal gene is carried on the X chromosome, such as hemophilia or Duchenne muscular dystrophy.

What is an indicator for performing a contraction stress test? A. Increased fetal movement and small for gestational age B. Maternal diabetes mellitus and postmaturity C. Adolescent pregnancy and poor prenatal care D. History of preterm labor and intrauterine growth restriction

B. Maternal diabetes mellitus and postmaturity Decreased fetal movement is an indicator for performing a contraction stress test; the size (small for gestational age) is not an indicator. Maternal diabetes mellitus and postmaturity are two indications for performing a contraction stress test. Although adolescent pregnancy and poor prenatal care are risk factors of poor fetal outcomes, they are not indicators for performing a contraction stress test. Intrauterine growth restriction is an indicator; but history of a previous stillbirth, not preterm labor, is the other indicator.

A pregnant woman in her first trimester with a history of epilepsy is transported to the hospital via ambulance after suffering a seizure in a restaurant. The nurse expects which health care provider orders to be included in the plan of care? (Select all that apply.) A. valproate (Depakote). B. Serum lab levels of medications. C. Abdominal ultrasounds. D. Prenatal vitamins with vitamin D. E. carbamazepine (Tegretol).

B. Serum lab levels of medications. C. Abdominal ultrasounds. D. Prenatal vitamins with vitamin D. Carbamazepine (Tegretol) and valproate (Depakote) should be avoided if possible during pregnancy, especially during the first trimester, because their use is associated with NTDs in the fetus. Checking lab levels of medications, performing abdominal ultrasounds to assess fetal growth, and taking prenatal vitamins with vitamin D are all expected interventions for a pregnant woman diagnosed with epilepsy.

A nonstress test (NST) is ordered on a pregnant women at 37 weeks gestation. What are the most appropriate teaching points to include when explaining the procedure to the patient? (Select all that apply) A. After 20 minutes, a nonreactive reading indicates the test is complete. B. Vibroacoustic stimulation may be used during the test. C. Drinking orange juice before the test is appropriate. D. A needle biopsy may be needed to stimulate contractions. E. Two sensors are placed on the abdomen to measure contractions and fetal heart tones.

B. Vibroacoustic stimulation may be used during the test. C. Drinking orange juice before the test is appropriate. E.Two sensors are placed on the abdomen to measure contractions and fetal heart tones. A nonreactive test requires further evaluation. The testing period is often extended, usually for an additional 20 minutes, with the expectation that the fetal sleep state will change and the test will become reactive. During this time vibroacoustic stimulation (see later discussion) may be used to stimulate fetal activity. Vibroacoustic stimulation is often used to stimulate fetal activity if the initial NST result is nonreactive and thus hopefully shortens the time required to complete the test (Greenberg, Druzin, and Gabbe, 2012). Care providers sometimes suggest that the woman drink orange juice or be given glucose to increase her blood sugar level and thereby stimulate fetal movements. Although this practice is common, there is no evidence that it increases fetal activity (Greenberg, Druzin, and Gabbe, 2012). A needle biopsy is not part of a NST. The FHR is recorded with a Doppler transducer, and a tocodynamometer is applied to detect uterine contractions or fetal movements. The tracing is observed for signs of fetal activity and a concurrent acceleration of FHR.

From 4% to 8% of pregnant women have asthma, making it one of the most common preexisting conditions of pregnancy. Severity of symptoms usually peaks: A. in the first trimester. B. between 24 to 36 weeks of gestation. C. during the last 4 weeks of pregnancy. D. immediately postpartum

B. between 24 to 36 weeks of gestation. Women often have few symptoms of asthma during the first trimester. The severity of symptoms peaks between 24 and 36 weeks of gestation. Asthma appears to be associated with intrauterine growth restriction and preterm birth. During the last 4 weeks of pregnancy symptoms often subside. The period between 24 and 36 weeks of pregnancy is associated with the greatest severity of symptoms. Issues have often resolved by the time the woman delivers.

A pregnant woman with cardiac disease is informed about signs of cardiac decompensation. She should be told that the earliest sign of decompensation is most often: A. orthopnea. B. decreasing energy levels. C. moist frequent cough and frothy sputum. D. crackles (rales) at the bases of the lungs on auscultation.

B. decreasing energy levels. Orthopnea is a finding that appears later when a failing heart reduces renal perfusion and fluid accumulates in the pulmonary interstitial space, leading to pulmonary edema. Decreasing energy level (fatigue) is an early finding of heart failure. Care must be taken to recognize it as a warning rather than a typical change of the third trimester. Cardiac decompensation is most likely to occur early in the third trimester, during childbirth, and during the first 48 hours following birth. A moist, frequent cough appears later when a failing heart reduces renal perfusion and fluid accumulates in the pulmonary interstitial space, leading to pulmonary edema. Crackles and rales appear later when a failing heart reduces renal perfusion and fluid accumulates in the pulmonary interstitial space, leading to pulmonary edema.

Diabetes in pregnancy puts the fetus at risk in several ways. Nurses should be aware that: A. with good control of maternal glucose levels, sudden and unexplained stillbirth is no longer a major concern. B. the most important cause of perinatal loss in diabetic pregnancy is congenital malformations. C. infants of mothers with diabetes have the same risks for respiratory distress syndrome because of the careful monitoring. D. at birth, the neonate of a diabetic mother is no longer at any greater risk.

B. the most important cause of perinatal loss in diabetic pregnancy is congenital malformations. Even with good control, sudden and unexplained stillbirth remains a major concern. Congenital malformations account for 30% to 50% of perinatal deaths. Infants of diabetic mothers are at increased risk for respiratory distress syndrome. The transition to extrauterine life often is marked by hypoglycemia and other metabolic abnormalities.

Several noted health risks are associated with menopause. These risks include all except: A) osteoporosis. B) coronary heart disease. C) breast cancer. D) obesity.

C) Breast Cancer Osteoporosis is a major health problem in the United States. It is associated with an increase in hip and vertebral fractures in postmenopausal women. A woman's risk of developing and dying of cardiovascular disease increases significantly after menopause. Breast cancer may be associated with the use of hormone replacement therapy for women who have a family history of breast cancer. Women tend to become more sedentary in midlife. The metabolic rate decreases after menopause, which may require an adjustment in lifestyle and eating patterns.

An effective relief measure for primary dysmenorrhea would be to: A) reduce physical activity level until menstruation ceases. B) begin taking prostaglandin synthesis inhibitors on the first day of the menstrual flow. C) decrease intake of salt and refined sugar about 1 week before menstruation is about to occur. D) use barrier methods rather than the oral contraceptive pill (OCP) for birth control.

C) decrease intake of salt and refined sugar about 1 week before menstruation is about to occur. Staying active is helpful since it facilitates menstrual flow and increases vasodilation to reduce ischemia. Prostaglandin inhibitors should be started a few days before the onset of menstruation. Decreasing intake of salt and refined sugar can reduce fluid retention. OCPs are beneficial in relieving primary dysmenorrhea as a result of inhibition of ovulation and prostaglandin synthesis.

Self-care instructions for a woman following a modified radical mastectomy would include that she: A) wears clothing with snug sleeves to support her affected arm. B) use depilatory creams instead of shaving the axilla of her affected arm. C) expect a decrease in sensation or tingling in her affected arm as her body heals. D) empty surgical drains once a day or every other day.

C) expect a decrease in sensation or tingling in her affected arm as her body heals. Loose clothing should be worn since tight clothing could impede circulation in the affected arm. The axilla of the affected arm should not be shaved nor should depilatory creams or strong deodorants be used. A decrease in sensation and tingling in the affected arm and in the incision are expected for weeks to months after the surgery. Drains should be emptied at least twice a day and more often if necessary.

A couple has been counseled for genetic anomalies. They ask you, "What is karyotyping?" Your best response is: A. "Karyotyping will reveal if the baby's lungs are mature." B. "Karyotyping will reveal if your baby will develop normally." C. "Karyotyping will provide information about the gender of the baby, and the number and structure of the chromosomes." D. "Karyotyping will detect any physical deformities the baby has."

C. "Karyotyping will provide information about the gender of the baby, and the number and structure of the chromosomes." The lecithin/sphingomyelin ratio, not karyotyping, reveals lung maturity. Although karyotyping can detect genetic anomalies, the range of normal is nondescriptive. Karyotyping provides genetic information, such as gender and chromosomal structure. Although karyotyping can detect genetic anomalies, not all such anomalies display obvious physical deformities. The term deformities is a nondescriptive word. Furthermore, physical anomalies may be present that are not detected by genetic studies (e.g., cardiac malformations).

A woman's cousin gave birth to an infant with a congenital heart anomaly. The woman asks the nurse when such anomalies occur during development. Which response by the nurse is most accurate? A. "We don't really know when such defects occur." B. "It depends on what caused the defect." C. "They occur between the third and fifth weeks of development." D. "They usually occur in the first 2 weeks of development."

C. "They occur between the third and fifth weeks of development." This is an inaccurate statement. Regardless of the cause, the heart is vulnerable during its period of development, the third to fifth weeks. The cardiovascular system is the first organ system to function in the developing human. Blood vessel and blood formation begins in the third week, and the heart is developmentally complete in the fifth week. This is an inaccurate statement.

The student nurse is giving a presentation about milestones in embryonic development. Which information should he or she include? A. At 8 weeks of gestation, primary lung and urethral buds appear. B. At 12 weeks of gestation, the vagina is open or the testes are in position for descent into the scrotum. C. At 20 weeks of age, the vernix caseosa and lanugo appear. D. At 24 weeks of age, the skin is smooth, and subcutaneous fat is beginning to collect.

C. At 20 weeks of age, the vernix caseosa and lanugo appear. The primary lung and urethral buds appear at 6 weeks of gestation. The vagina is open or the testes are in position for descent into the scrotum at 16 weeks. Two milestones that occur at 20 weeks are the appearance of the vernix caseosa and lanugo. The appearance of smooth skin occurs at 28 weeks, and subcutaneous fat begins to collect at 30 to 31 weeks.

A nurse is providing genetic counseling for an expectant couple who already have a child with trisomy 18. The nurse should: A. Tell the couple they need to have an abortion within 2 to 3 weeks. B. Explain that the fetus has a 50% chance of having the disorder. C. Discuss options with the couple, including amniocentesis to determine whether the fetus is affected. D. Refer the couple to a psychologist for emotional support.

C. Discuss options with the couple, including amniocentesis to determine whether the fetus is affected. The couple should be given information about the likelihood of having another baby with this disorder so that they can make an informed decision. A genetic counselor is the best source for determining genetic probability ratios. Genetic testing, including amniocentesis, would need to be performed to determine whether the fetus is affected. The couple eventually may need emotional support, but the status of the pregnancy must be determined first.

A woman at 35 weeks of gestation has had an amniocentesis. The results reveal that surface-active phospholipids are present in the amniotic fluid. The nurse is aware that this finding indicates: A The fetus is at risk for Down syndrome. B. The woman is at high risk for developing preterm labor. C. Lung maturity. D. Meconium is present in the amniotic fluid.

C. Lung maturity. The presence of surface-active phospholipids is not an indication of Down syndrome. This result reveals the fetal lungs are mature and in no way indicates risk for preterm labor. The detection of the presence of pulmonary surfactants, surface-active phospholipids, in amniotic fluid has been used to determine fetal lung maturity, or the ability of the lungs to function after birth. This occurs at approximately 35 weeks of gestation. Meconium should not be present in the amniotic fluid.

Maternal and neonatal risks associated with gestational diabetes mellitus are: A. maternal premature rupture of membranes and neonatal sepsis. B. maternal hyperemesis and neonatal low birth weight. C. Maternal preeclampsia and fetal macrosomia. D. maternal placenta previa and fetal prematurity.

C. Maternal preeclampsia and fetal macrosomnia Premature rupture of membranes and neonatal sepsis are not risks associated with gestational diabetes. Hyperemesis is not seen with gestational diabetes, nor is there an association with low birth weight of the infant. Women with gestational diabetes have twice the risk of developing hypertensive disorders such as preeclampsia, and the baby usually has macrosomia. Placental previa and subsequent prematurity of the neonate are not risks associated with gestational diabetes.

During pregnancy, alcohol withdrawal may be treated using: A. disulfiram (Antabuse). B. corticosteroids. C. benzodiazepines. E. aminophylline.

C. benzodiazepines. Disulfiram is contraindicated in pregnancy because it is teratogenic. Corticosteroids are not used to treat alcohol withdrawal. Symptoms that occur during alcohol withdrawal can be managed with short-acting barbiturates or benzodiazepines. Aminophylline is not used to treat alcohol withdrawal.

A woman who is at 36 weeks of gestation is having a nonstress test. Which statement by the woman would indicate a correct understanding of the test? A. "I will need to have a full bladder for the test to be done accurately." B. "I should have my husband drive me home after the test because I may be nauseous." C. "This test will help to determine if the baby has Down syndrome or a neural tube defect." D."This test will observe for fetal activity and an acceleration of the fetal heart rate to determine the well-being of the baby."

D. "This test will observe for fetal activity and an acceleration of the fetal heart rate to determine the well-being of the baby." An ultrasound is the test that requires a full bladder. An amniocentesis would be the test that a pregnant woman should be driven home afterward. A maternal alpha-fetoprotein test is used in conjunction with unconjugated estriol levels, and human chorionic gonadotropin helps to determine Down syndrome. The nonstress test is one of the most widely used techniques to determine fetal well-being and is accomplished by monitoring fetal heart rate in conjunction with fetal activity and movements.

The health care provider has ordered a magnetic resonance imaging (MRI) study to be done on a pregnant patient to evaluate fetal structure and growth. The nurse should include which instructions when preparing the patient for this test? (Select all that apply.) A. A lead apron must be worn during the test. B. A full bladder is required prior to the test. C. An intravenous line must be inserted before the test. D. Jewelry must be removed before the test. E. Remain still throughout the test.

D. Jewelry must be removed before the test. E. Remain still throughout the test. Magnetic resonance imaging (MRI) is a noninvasive radiologic technique used for obstetric and gynecologic diagnosis. Similar to computed tomography (CT), MRI provides excellent pictures of soft tissue. Unlike CT, ionizing radiation is not used. Therefore vascular structures within the body can be visualized and evaluated without injecting an iodinated contrast medium, thus eliminating any known biologic risk. Similar to sonography, MRI is noninvasive and can provide images in multiple planes, but no interference occurs from skeletal, fatty, or gas-filled structures, and imaging of deep pelvic structures does not require a full bladder. The woman is placed on a table in the supine position and moved into the bore of the main magnet, which is similar in appearance to a CT scanner. Depending on the reason for the study, the procedure may take from 20 to 60 minutes, during which time the woman must be perfectly still except for short respites.

15. Nurses should be aware of the strengths and limitations of various biochemical assessments during pregnancy, including that: A. chorionic villus sampling (CVS) is becoming more popular because it provides early diagnosis. B. screening for maternal serum alpha-fetoprotein (MSAFP) levels is recommended only for women at risk for neural tube defects. C. percutaneous umbilical blood sampling (PUBS) is one of the quad-screen tests for Down syndrome. D. MSAFP is a screening tool only; it identifies candidates for more definitive procedures.

D. MSAFP is a screening tool only; it identifies candidates for more definitive procedures. CVS does provide a rapid result, but it is declining in popularity because of advances in noninvasive screening techniques. MSAFP screening is recommended for all pregnant women. MSAFP, not PUBS, is part of the quad-screen tests for Down syndrome. This is correct. MSAFP is a screening tool, not a diagnostic tool. Further diagnostic testing is indicated after an abnormal MSAFP.

With regard to the estimation and interpretation of the recurrence of risks for genetic disorders, nurses should be aware that: A. With a dominant disorder, the likelihood of the second child also having the condition is 100%. B. An autosomal recessive disease carries a one in eight risk of the second child also having the disorder. C. Disorders involving maternal ingestion of drugs carry a one in four chance of being repeated in the second child. D. The risk factor remains the same no matter how many affected children are already in the family.

D. The risk factor remains the same no matter how many affected children are already in the family. In a dominant disorder, the likelihood of recurrence in subsequent children is 50% (one in two). An autosomal recessive disease carries a one in four chance of recurrence. Subsequent children would be at risk only if the mother continued to use drugs; the rate of risk would be difficult to calculate. Each pregnancy is an independent event. The risk factor (e.g., one in two, one in four) remains the same for each child, no matter how many children are born to the family.

During a physical assessment of an at-risk client, the nurse notes generalized edema, crackles at the base of the lungs, and some pulse irregularity. These are most likely signs of: A. euglycemia. B. rheumatic fever. C. pneumonia. D. cardiac decompensation.

D. cardiac decompensation. Euglycemia is a condition of normal glucose levels. These symptoms indicate cardiac decompensation. Rheumatic fever can cause heart problems, but it does not present with these symptoms, which indicate cardiac decompensation. Pneumonia is an inflammation of the lungs and would not likely generate these symptoms, which indicate cardiac decompensation. Symptoms of cardiac decompensation may appear abruptly or gradually.

In planning for the care of a 30-year-old woman with pregestational diabetes, the nurse recognizes that the most important factor affecting pregnancy outcome is the: A. mother's age. B. number of years since diabetes was diagnosed. C. amount of insulin required prenatally. D. degree of glycemic control during pregnancy.

D. degree of glycemic control during pregnancy. Although advanced maternal age may pose some health risks, for the woman with pregestational diabetes the most important factor remains the degree of glycemic control during pregnancy. The number of years since diagnosis is not as relevant to outcomes as the degree of glycemic control. The key to reducing risk in the pregestational diabetic woman is not the amount of insulin required but rather the level of glycemic control. Women with excellent glucose control and no blood vessel disease should have good pregnancy outcomes.

A pregnant woman at 28 weeks of gestation has been diagnosed with gestational diabetes. The nurse caring for this client understands that: A. oral hypoglycemic agents can be used if the woman is reluctant to give herself insulin. B. dietary modifications and insulin are both required for adequate treatment. C. glucose levels are monitored by testing urine 4r times a day and at bedtime. D. dietary management involves distributing nutrient requirements over three meals and two or three snacks.

D. dietary management involves distributing nutrient requirements over three meals and two or three snacks. Oral hypoglycemic agents can be harmful to the fetus and less effective than insulin in achieving tight glucose control. In some women gestational diabetes can be controlled with dietary modifications alone. Blood, not urine, glucose levels are monitored several times a day. Urine is tested for ketone content; results should be negative. Small frequent meals over a 24-hour period help decrease the risk for hypoglycemia and ketoacidosis.

3 Repetitive transvaginal ultrasounds and measurement of human chorionic gonadotropin (hCG) and progesterone levels may be performed to determine if the fetus is alive and within the uterus. If the pregnancy is lost , the woman should be guided through the grieving process. D&C is not considered until signs of the progress to an inevitable abortion are noted or the contents are expelled and incomplete. Bed rest is recommended for 48 hours initially. Telling the woman that she can get pregnant again soon is not a therapeutic response because it discounts the importance of this pregnancy.

Signs of a threatened abortion (miscarriage) are noted in a woman at 8 weeks of gestation. What is an appropriate management approach for this type of abortion? 1 Prepare the woman for a dilation and curettage (D&C). 2 Place the woman on bed rest for at least 1 week and reevaluate. 3 Prepare the woman for an ultrasound and bloodwork. 4 Comfort the woman by telling her that if she loses this baby, she may attempt to get pregnant again in 1 month.

2 Pain is absent with placenta previa but may be agonizing with abruptio placentae. Bleeding may be present in varying degrees for both placental conditions. Uterine activity may be present with both placental conditions. Cramping is a form of uterine activity that may be present in both placental conditions.

The most prevalent clinical manifestation of abruptio placentae (as opposed to placenta previa) is: 1 bleeding. 2 intense abdominal pain. 3 uterine activity. 4 cramping

1 The antibiotic therapy kills normal flora in the genitourinary tract, as well as pathologic organisms. Therefore the nurse instructs the patient to include yogurt, cheese, and milk in daily diet because they contain active acidophilus cultures. Folic acid should not be avoided, because it may affect the fetal development. Vitamins C and E supplementation is usually included in the diet to treat preeclampsia in a patient. Dietary fat is reduced in patients with cholecystitis or cholelithiasis, because it may cause epigastric pain.

The nurse is caring for a pregnant patient who is receiving antibiotic therapy to treat a urinary tract infection (UTI). Which dietary changes does the nurse suggest for the pregnant patient who is receiving antibiotic therapy for UTI? 1 "Include yogurt, cheese, and milk in your diet." 2 "Avoid folic acid supplements until the end of therapy." 3 "Include vitamins C and E supplementation in your diet." 4 "Reduce your dietary fat intake by 40 to 50 g per day.

1, 3, 4 Decreased urinary output and irritability are signs of severe eclampsia. Ankle clonus and epigastric pain are signs of severe eclampsia. Platelet count of less than 100,000/mm3 and visual problems are signs of severe preeclampsia. A transient headache and +1 proteinuria are signs of preeclampsia and should be monitored.Seizure activity and hyperreflexia are signs of eclampsia. Test-Taking Tip: Do not worry if you select the same numbered answer repeatedly, because there usually is no pattern to the answers.

The nurse is caring for a woman who is at 24 weeks of gestation with suspected severe preeclampsia. Which signs and symptoms should the nurse expect to observe? Select all that apply. 1 Decreased urinary output and irritability 2 Transient headache and +1 proteinuria 3 Ankle clonus and epigastric pain 4 Platelet count of less than 100,000/mm3 and visual problems 5 Seizure activity and hypotension

3 A woman who has experienced a miscarriage should be advised to eat foods that are high in iron and protein to help replenish her body after the loss. After a miscarriage, a woman may experience mood swings and depression from the reduction of hormones and the grieving process. Sexual intercourse should be avoided for 2 weeks or until the bleeding has stopped and should avoid pregnancy for 2 months. The woman should not experience bright red, heavy, profuse bleeding; this should be reported to the health care provider.

The nurse is preparing to discharge a 30-year-old woman who has experienced a miscarriage at 10 weeks of gestation. Which statement by the woman indicates a correct understanding of the discharge instructions? 1 "I will not experience mood swings since I was only at 10 weeks of gestation." 2 "I will avoid sexual intercourse for 6 weeks and pregnancy for 6 months." 3 "I should eat foods that are high in iron and protein to help my body heal." 4 "I should expect the bleeding to be heavy and bright red for at least 1 week."

2 Concurrent use of nifedipine (Adalat) and magnesium sulfate can result in skeletal muscle blockade in the patient. Therefore the nurse needs to report immediately to the primary health care provider (PHP) and obtain a prescription for a change of drug. The nurse assesses the patient's renal function to determine the risk for toxicity after administering any drug. However, it is not a priority in this case. Reducing the nifedipine (Adalat) dose is not likely to prevent the drug interaction in the patient. The nurse does not administer both drugs simultaneously because it may be harmful for the patient.

The nurse observes that a pregnant patient with gestational hypertension who is on magnesium sulfate therapy is prescribed nifedipine (Adalat). What action does the nurse take? 1Evaluates the patient's renal function test 2Obtains a prescription for a change of drug 3Reduces the nifedipine (Adalat) dose by 50% 4Administers both medications simultaneously

3 Electronic fetal monitoring reflects fetal cardiac responses to hypoxia and hypoperfusion and helps to assess maternal status after a trauma. The D-dimer blood test is used to rule out the presence of a thrombus. The KB test is used to evaluate transplacental hemorrhage. Electrocardiogram reading is more useful to assess the cardiac functions in nonpregnant cardiac patients.

The nurse observes that maternal hypotension has decreased uterine and fetal perfusion in a pregnant patient. What does the nurse need to assess further to understand the maternal status? 1 D-dimer blood test 2 Kleihauer-Betke (KB) test 3 Electronic fetal monitoring 4 Electrocardiogram reading

1 High β-hCG levels indicate that the abdominal pregnancy is not yet dissolved. Therefore the nurse advises the patient to avoid sexual activity until the β-hCG levels drop and the pregnancy is dissolved completely. If the patient engages in vaginal intercourse, the pelvic pressure may rupture the mass and cause pain. Abdominal pregnancy increases the chances of infertility or recurrent ectopic pregnancy in patients. However, the nurse need not instruct the patient to avoid further pregnancy, because it may increase the feelings of sadness and guilt in the patient. The nurse encourages the patient to share feelings of guilt or sadness related to pregnancy loss. Folic acid is contraindicated with methotrexate therapy, because it may exacerbate ectopic rupture.

The quantitative human chorionic gonadotropin (β-hCG) levels are high in a patient who is on methotrexate therapy for dissolving abdominal pregnancy. Which instruction does the nurse give to this patient? 1 "Avoid sexual activity." 2 "Avoid next pregnancy." 3 "Avoid feeling sad and low." 4 "Take folic acid without fail."

2 Choking is often relieved in patients by administering abdominal thrusts. However if the patient is in the third trimester of pregnancy, chest thrusts are administered to prevent injury to the uterus. Administering anesthesia or positioning the patient onto one side will not help dislodge the object and relieve choking. The nurse needs to place a towel under the hips to displace the uterus while administering cardiopulmonary resuscitation (CPR).

What action does the nurse take to relieve choking in a pregnant patient who is in the third trimester? 1 Administering anesthesia 2 Administering chest thrusts 3 Placing a towel under the hips 4 Positioning the patient onto one side

2, 3, 4, 5 Chromosomal abnormalities account for 50% of all early pregnancy losses. Endocrine imbalance is caused by luteal phase defects, hypothyroidism, and diabetes mellitus in pregnant patients and results in miscarriage. Antiphospholipid antibodies also increase the chances of miscarriage in pregnant patients. Premature dilation of the cervix may cause a second-trimester loss and is usually seen in patients between 12 and 20 weeks' gestation.

What are the possible causes of miscarriage during early pregnancy? Select all that apply. 1 Premature dilation of cervix 2 Chromosomal abnormalities 3 Endocrine imbalance 4 Hypothyroidism 5 Antiphospholipid antibodies

4 Ergonovine (Methergine) is an ergot product, which is administered to contract the uterus when there is excessive bleeding after suction curettage. Nifedipine (Procardia) is prescribed for gestational hypertension or severe preeclampsia. Methyldopa (Aldomet) is an antihypertensive medication indicated for pregnant patients with hypertension. Hydralazine (Apresoline) is also an antihypertensive medication used for treating hypertension intrapartum.

What does the nurse administer to a patient if there is excessive bleeding after suction curettage? 1 Nifedipine (Procardia) 2 Methyldopa (Aldomet) 3 Hydralazine (Apresoline) 4 Ergonovine (Methergine)

3 Phenazopyridine (Pyridium) colors the tears orange. Therefore the nurse instructs the patient to avoid wearing contact lenses. Sweet foods are avoided in patients with diabetes mellitus, because they can cause fluctuating glucose levels, which may harm the fetus. Exposure to sunlight is avoided when the patient is receiving methotrexate therapy, because it causes photosensitivity. Oral fluids are restricted in patients who are at risk for pulmonary edema.

What does the nurse advise a pregnant patient who is prescribed phenazopyridine (Pyridium) for cystitis? 1 "Avoid sweet foods in diet." 2 "Limit exposure to sunlight." 3 "Do not wear contact lenses." 4 "Restrict oral fluids to 125 mL per hour."

1, 2, 4, 5 Proteinuria indicates hypertension in a pregnant patient. Proteinuria is concentration ≥300 mg/24 hours in a 24-hour urine collection. The nurse needs to assess the patient for epigastric pain because it indicates severe preeclampsia. Hypertension is likely to cause edema or swollen ankles as a result of greater hydrostatic pressure in the lower parts of the body. Therefore the nurse needs to assess the patient for the presence of edema. Accurate measurement of BP will help detect the presence of any hypertensive disorder. A systolic BP greater than 140 mm Hg or a diastolic BP greater than 90 mm Hg will indicate hypertension. Placenta previa is a condition wherein the placenta is implanted in the lower uterine segment covering the cervix, which causes bleeding when the cervix dilates.

What does the nurse assess to detect the presence of a hypertensive disorder in a pregnant patient? Select all that apply. 1 Proteinuria 2 Epigastric pain 3 Placenta previa 4 Presence of edema 5 Blood pressure (BP)

2, 3, 5 Activity is restricted in patients with preeclampsia, so it is necessary to provide diversionary activities to such patients to prevent boredom. The nurse encourages the patient to increase fluid intake to enhance renal perfusion and bowel function. The nurse can suggest Internet-based support groups to reduce boredom and stress in the patient. Patients need to restrict activity, but complete bed rest is not advised because it may cause cardiovascular deconditioning, muscle atrophy, and psychological stress. The patient needs to include adequate zinc and sodium in the diet for proper fetal development.

What does the nurse include in the plan of care of a pregnant patient with mild preeclampsia? Select all that apply. 1 Ensure prolonged bed rest. 2 Provide diversionary activities. 3 Encourage the intake of more fluids. 4 Restrict sodium and zinc in the diet. 5 Refer to Internet-based support group

3 Preeclampsia can affect the fetus and may cause fetal growth restrictions, decreased amniotic fluid volume, abnormal fetal oxygenation, low birth weight, and preterm birth. Therefore the fetal movements need to be evaluated daily. Patients with mild preeclampsia can be managed at home effectively and need not be hospitalized. Nonstress testing is performed once or twice per week to determine fetal well-being. Patients need to restrict activity, but complete bed rest is not advised because it may cause cardiovascular deconditioning, muscle atrophy, and psychological stress.

What instruction does the nurse provide to a pregnant patient with mild preeclampsia? 1 "You need to be hospitalized for fetal evaluation." 2 "Nonstress testing can be done once every month." 3 "Fetal movement counts need to be evaluated daily." 4 "Take complete bed rest during the entire pregnancy."

1, 2, 3 An ectopic pregnancy is indicated when β-hCG levels are >1500 milli-international units/mL but no intrauterine pregnancy is seen on the transvaginal ultrasound. A transvaginal ultrasound is repeated to verify if the pregnancy is inside the uterus. A progesterone level <5 ng/mL indicates ectopic pregnancy. Thyroid test reports need to be evaluated in case the patient has hyperemesis gravidarum, as hyperthyroidism is associated with this disorder. The KB test is used to determine transplacental hemorrhage.

Which clinical reports does the nurse evaluate to identify ectopic pregnancy in a patient? Select all that apply. 1 Quantitative human chorionic gonadotropin (β-hCG) levels 2 Transvaginal ultrasound 3 Progesterone level 4 Thyroid test reports 5 Kleihauer-Betke (KB) test

1 Severe preeclampsia may cause seizure activity or eclampsia in the patient, which is treated with magnesium sulfate. Magnesium sulfate is not administered for renal dysfunction and can cause magnesium toxicity in the patient. Pulmonary enema can be prevented by restricting the patient's fluid intake to 125 mL/hr. Increasing magnesium toxicity can cause low BP in the patient.

Which condition in a pregnant patient with severe preeclampsia is an indication for administering magnesium sulfate? 1 Seizure activity 2 Renal dysfunction 3 Pulmonary edema 4 Low blood pressure (BP)

1 Preeclampsia is a condition in which patients develop hypertension and proteinuria after 20 weeks' gestation. It can be diagnosed if uterine artery Doppler measurements in the second trimester of pregnancy are abnormal. HELLP syndrome is characterized by hemolysis (H), elevated liver enzymes (EL), and low platelet count (LP) in a patient with preeclampsia. Molar pregnancy refers to the growth of the placental trophoblast due to abnormal fertilization. Gestational hypertension is a condition in which hypertension develops in a patient after 20 weeks' gestation.

Which condition is seen in a pregnant patient if uterine artery Doppler measurements in the second trimester of pregnancy are abnormal? 1Preeclampsia 2HELLP syndrome 3Molar pregnancy 4Gestational hypertension

1, 2, 3, 4 Current theories consider that genetic abnormalities and dietary deficiencies can result in preeclampsia. Abnormal trophoblast invasion causes fetal hypoxia and results in maternal hypertension. Cardiovascular changes stimulate the inflammatory system and result in preeclampsia in the pregnant patient. Maternal hypertension, and not hypotension, after 20 weeks' gestation is known as preeclampsia.

Which conditions during pregnancy can result in preeclampsia in the patient? Select all that apply. 1 Genetic abnormalities 2 Dietary deficiencies 3 Abnormal trophoblast invasion 4 Cardiovascular changes 5 Maternal hypotension

4 In an ectopic pregnancy, the risk for fetal deformity is high because of the pressure deformities caused by oligohydramnios. There may be facial or cranial asymmetry, various joint deformities, limb deficiency, and central nervous system (CNS) anomalies. Miscarriage is not likely to happen in an ectopic pregnancy. Instead, the patient is at risk for pregnancy-related death resulting from ectopic rupture. Fetal anemia is a risk associated with placenta previa. Preterm birth is not possible because the pregnancy is dissolved when it is diagnosed or a surgery is performed to remove the fetus.

Which fetal risk is associated with an ectopic pregnancy? 1 Miscarriage 2 Fetal anemia 3 Preterm birth 4 Fetal deformity

1, 2, 4 Chronic hypertension refers to hypertension that developed in the pregnant patient before 20 weeks' gestation. Preeclampsia refers to hypertension and proteinuria that develops after 20 weeks' gestation. Eclampsia is the onset of seizure activity in a pregnant patient with preeclampsia. Gestational hypertension is the onset of hypertension after 20 weeks' gestation. Gestational trophoblastic disease and hyperemesis gravidarum are not hypertensive disorders. Gestational trophoblastic disease refers to a disorder without a viable fetus that is caused by abnormal fertilization. Hyperemesis gravidarum is excessive vomiting during pregnancy that may result in weight loss and electrolyte imbalance.

Which hypertensive disorders can occur during pregnancy? Select all that apply. 1 Chronic hypertension 2 Preeclampsia-eclampsia 3 Hyperemesis gravidarum 4 Gestational hypertension 5 Gestational trophoblastic disease

1 The nurse advises the patient to discontinue folic acid supplements as they interact with methotrexate and may exacerbate ectopic rupture in the patient. Exposure to sunlight is avoided as the therapy makes the patient photosensitive. Analgesics stronger than acetaminophen are avoided, because they may mask symptoms of tubal rupture. Vaginal intercourse is avoided until the pregnancy is dissolved completely.

Which instructions does the nurse give to a patient who is prescribed methotrexate therapy for dissolving the tubal pregnancy? 1 "Discontinue folic acid supplements." 2 "Get adequate exposure to sunlight." 3 "Take stronger analgesics for severe pain." 4 "Vaginal intercourse is safe during the therapy."

2 Pelvic CT scanning helps visualize extraperitoneal and retroperitoneal structures and the genitourinary tract. The nurse needs to prepare the patient for cesarean birth if there is no evidence of a maternal pulse. Ultrasound examination is not as effective as electronic fetal monitoring for determining placental abruption in the patient after the trauma. Therefore the nurse prepares the patient for a CT scan after a severe abdominal trauma. The nurse needs to administer Rho(D) immunoglobulin in an Rh-negative pregnant trauma patient. This helps protect the patient from isoimmunization.

Which intervention does the nurse implement for a patient immediately after a severe abdominal trauma? 1 Prep the patient for cesarean birth. 2 Send the patient for pelvic computed tomography (CT) scanning. 3 Provide fluids to the patient as part of the protocol for ultrasound examination. 4 Prepare to administer Rho(D) immunoglobulin.

3 Pulmonary edema may be seen in patients with severe preeclampsia. Therefore the nurse needs to restrict total intravenous (I.V.) and oral fluids to 125 mL/hr. FHR monitoring helps assess any fetal complications. The patient is placed on bed rest in a darkened environment to prevent stress. Magnesium sulfate is administered to prevent eclamptic seizures.

Which intervention will help prevent the risk of pulmonary edema in a pregnant patient with severe preeclampsia? 1 Assess fetal heart rate (FHR) abnormalities regularly. 2 Place the patient on bed rest in a darkened environment. 3 Restrict total intravenous (I.V.) and oral fluids to 125 mL/hr. 4 Ensure that magnesium sulfate is administered as prescribed.

4 The nurse administers the prescribed magnesium sulfate to the patient to prevent eclamptic seizures. I.V. oral fluids are indicated when there is severe dehydration in the patient. It is important to provide diversionary activities during bed rest, but it is secondary in this case. A patient who has experienced a multisystem trauma is prepared for cesarean delivery if there is no evidence of a maternal pulse, which increases the chance of maternal survival.

Which is a priority nursing action when a pregnant patient with severe gestational hypertension is admitted to the health care facility? 1 Prepare the patient for cesarean delivery. 2 Administer intravenous (I.V.) and oral fluids. 3 Provide diversionary activities during bed rest. 4 Administer the prescribed magnesium sulfate.

2 In the case of an incomplete miscarriage, sometimes there is heavy bleeding and excessive cramping and some part of fetal tissue remains in the uterus. Therefore the nurse needs to prepare the patient for dilation and curettage for the removal of the fetal tissue. Expectant management is initiated if the pregnancy continues after a threatened miscarriage. Oxytocin (Pitocin) is administered to prevent hemorrhage after evacuation of the uterus. Ergonovine (Methergine) is administered to contract the uterus.

Which is an important nursing intervention when a patient has an incomplete miscarriage with heavy bleeding? 1 Initiate expectant management at once. 2 Prepare the patient for dilation and curettage. 3 Administer the prescribed oxytocin (Pitocin). 4 Obtain a prescription for ergonovine (Methergine).

3. Which symptom described by a patient is characteristic of premenstrual syndrome (PMS)? a. "I feel irritable and moody a week before my period is supposed to start." b. "I have lower abdominal pain beginning the third day of my menstrual period." c. "I have nausea and headaches after my period starts, and they last 2 to 3 days." d. "I have abdominal bloating and breast pain after a couple days of my period."

a. "I feel irritable and moody a week before my period is supposed to start."

36. When the nurse is alone with a battered patient, the patient seems extremely anxious and says, "It was all my fault. The house was so messy when he got home and I know he hates that." The best response by the nurse is: a. "No one deserves to be hurt. It's not your fault. How can I help you?" b. "What else do you do that makes him angry enough to hurt you?" c. "He will never find out what we talk about. Don't worry. We're here to help you." d. "You have to remember that he is frustrated and angry so he takes it out on you."

a. "No one deserves to be hurt. It's not your fault. How can I help you?"

21. A pregnant woman's biophysical profile score is 8. She asks the nurse to explain the results. The nurse's best response is: a. "The test results are within normal limits." b. "Immediate delivery by cesarean birth is being considered." c. "Further testing will be performed to determine the meaning of this score." d. "An obstetric specialist will evaluate the results of this profile and, within the next week, will inform you of your options regarding delivery."

a. "The test results are within normal limits."

16. Which statement would indicate that the client requires additional instruction about breast self-examination? a. "Yellow discharge from my nipple is normal if I'm having my period." b. "I should check my breasts at the same time each month, like after my period." c. "I should also feel in my armpit area while performing my breast examination." d. "I should check each breast in a set way, such as in a circular motion."

a. "Yellow discharge from my nipple is normal if I'm having my period."

A woman asks the nurse, "What protects my baby's umbilical cord from being squashed while the baby's inside of me?" The nurse's best response is: a. "Your baby's umbilical cord is surrounded by connective tissue called Wharton jelly, which prevents compression of the blood vessels and ensures continued nourishment of your baby." b. "Your baby's umbilical floats around in blood anyway." c. "You don't need to worry about things like that." d. "The umbilical cord is a group of blood vessels that are very well protected by the placenta."

a. "Your baby's umbilical cord is surrounded by connective tissue called Wharton jelly, which prevents compression of the blood vessels and ensures continued nourishment of your baby." "Your baby's umbilical cord is surrounded by connective tissue called Wharton jelly, which prevents compression of the blood vessels and ensures continued nourishment of your baby" is the most appropriate response. "Your baby's umbilical floats around in blood anyway" is inaccurate. "You don't need to worry about things like that" is not appropriate response. It negates the client's need for teaching and discounts her feelings. The placenta does not protect the umbilical cord. The cord is protected by the surrounding Wharton jelly.

8. A client asks her nurse, "My doctor told me that he is concerned with the grade of my placenta because I am overdue. What does that mean?" The best response by the nurse is: a. "Your placenta changes as your pregnancy progresses, and it is given a score that indicates the amount of calcium deposits it has. The more calcium deposits, the higher the grade, or number, that is assigned to the placenta. It also means that less blood and oxygen can be delivered to your baby." b. "Your placenta isn't working properly, and your baby is in danger." c. "This means that we will need to perform an amniocentesis to detect if you have any placental damage." d. "Don't worry about it. Everything is fine."

a. "Your placenta changes as your pregnancy progresses, and it is given a score that indicates the amount of calcium deposits it has. The more calcium deposits, the higher the grade, or number, that is assigned to the placenta. It also means that less blood and oxygen can be delivered to your baby."

5. Because of the effect of cyclic ovarian changes on the breast, the best time for breast self-examination (BSE) is: a. 5 to 7 days after menses ceases. b. Day 1 of the endometrial cycle. c. Midmenstrual cycle. d. Any time during a shower or bath.

a. 5 to 7 days after menses ceases

12. A newly graduated nurse is attempting to understand the reason for increasing health care spending in the United States. Her research finds that these costs are much higher compared with other developed countries as a result of: a. A higher rate of obesity among pregnant women. b. Limited access to technology. c. Increased usage of health care services along with lower prices. d. Homogeneity of the population.

a. A higher rate of obesity among pregnant women.

17. Which statement about pregnancy is accurate? a. A normal pregnancy lasts about 10 lunar months. b. A trimester is one third of a year. c. The prenatal period extends from fertilization to conception. d. The estimated date of confinement (EDC) is how long the mother will have to be bedridden after birth.

a. A normal pregnancy lasts about 10 lunar months.

2. Prenatal testing for human immunodeficiency virus (HIV) is recommended for: a. All women, regardless of risk factors. b. A woman who has had more than one sexual partner. c. A woman who has had a sexually transmitted infection. d. A woman who is monogamous with her partner.

a. All women, regardless of risk factors.

20. A woman has a thick, white, lumpy, cottage cheese-like discharge, with patches on her labia and in her vagina. She complains of intense pruritus. The nurse practitioner would order which preparation for treatment? a. Fluconazole b. Tetracycline c. Clindamycin d. Acyclovir

a. Fluconazole

A key finding from the Human Genome Project is: a. Approximately 20,000 to 25,000 genes make up the genome. b. All human beings are 80.99% identical at the DNA level. c. Human genes produce only one protein per gene; other mammals produce three proteins per gene. d. Single gene testing will become a standardized test for all pregnant clients in the future.

a. Approximately 20,000 to 25,000 genes make up the genome. Approximately 20,000 to 25,000 genes make up the human genome; this is only twice as many as make up the genomes of roundworms and flies. Human beings are 99.9% identical at the DNA level. Most human genes produce at least three proteins. Single gene testing (e.g., alpha-fetoprotein) is already standardized for prenatal care.

23. On vaginal examination of a 30-year-old woman, the nurse documents the following findings: profuse, thin, grayish white vaginal discharge with a "fishy" odor; complaint of pruritus. On the basis of these findings, the nurse suspects that this woman has: a. Bacterial vaginosis (BV). b. Candidiasis. c. Trichomoniasis. d. Gonorrhea.

a. Bacterial vaginosis (BV)

18. The nurse caring for a pregnant client should be aware that the U.S. birth rate shows which trend? a. Births to unmarried women are more likely to have less favorable outcomes. b. Birth rates for women 40 to 44 years old are beginning to decline. c. Cigarette smoking among pregnant women continues to increase. d. The rates of maternal death owing to racial disparity are elevated in the United States.

a. Births to unmarried women are more likely to have less favorable outcomes.

The nurse is assessing the knowledge of new parents with a child born with maple syrup urine disease (MSUD). This is an autosomal recessive inherited disorder, which means that: a. Both genes of a pair must be abnormal for the disorder to be expressed. b. Only one copy of the abnormal gene is required for the disorder to be expressed. c. The disorder occurs in males and heterozygous females. d. The disorder is carried on the X chromosome.

a. Both genes of a pair must be abnormal for the disorder to be expressed. MSUD is a type of autosomal recessive inheritance disorder in which both genes of a pair must be abnormal for the disorder to be expressed. MSUD is not an X-linked dominant or recessive disorder or an autosomal dominant inheritance disorder.

29. With regard to medications, herbs, shots, and other substances normally encountered by pregnant women, the maternity nurse should be aware that: a. Both prescription and over-the-counter (OTC) drugs that otherwise are harmless can be made hazardous by metabolic deficiencies of the fetus. b. The greatest danger of drug-caused developmental deficits in the fetus is seen in the final trimester. c. Killed-virus vaccines (e.g., tetanus) should not be given during pregnancy, but live-virus vaccines (e.g., measles) are permissible. d. No convincing evidence exists that secondhand smoke is potentially dangerous to the fetus.

a. Both prescription and over-the-counter (OTC) drugs that otherwise are harmless can be made hazardous by metabolic deficiencies of the fetus.

2. The uterus is a muscular, pear-shaped organ that is responsible for: a. Cyclic menstruation. b. Sex hormone production. c. Fertilization. d. Sexual arousal.

a. Cyclic menstruation.

1. When providing care for a pregnant woman, the nurse should be aware that one of the most frequently reported maternal medical risk factors is: a. Diabetes mellitus. b. Mitral valve prolapse (MVP). c. Chronic hypertension. d. Anemia.

a. Diabetes mellitus.

39. Fibrocystic changes in the breast most often appear in women in their 20s and 30s. The etiology is unknown, but it may be an imbalance of estrogen and progesterone. The nurse who cares for this client should be aware that treatment modalities are conservative. One proven modality that may provide relief is: a. Diuretic administration. b. Including caffeine daily in the diet. c. Increased vitamin C supplementation. d. Application of cold packs to the breast as necessary.

a. Diuretic administration.

3. The nurse sees a woman for the first time when she is 30 weeks pregnant. The woman has smoked throughout the pregnancy, and fundal height measurements now are suggestive of growth restriction in the fetus. In addition to ultrasound to measure fetal size, what other tool would be useful in confirming the diagnosis? a. Doppler blood flow analysis b. Contraction stress test (CST) c. Amniocentesis d. Daily fetal movement counts

a. Doppler blood flow analysis Doppler blood flow analysis allows the examiner to study the blood flow noninvasively in the fetus and the placenta. It is a helpful tool in the management of high-risk pregnancies because of intrauterine growth restriction (IUGR), diabetes mellitus, multiple fetuses, or preterm labor. Because of the potential risk of inducing labor and causing fetal distress, a CST is not performed on a woman whose fetus is preterm. Indications for an amniocentesis include diagnosis of genetic disorders or congenital anomalies, assessment of pulmonary maturity, and the diagnosis of fetal hemolytic disease, not IUGR. Fetal kick count monitoring is performed to monitor the fetus in pregnancies complicated by conditions that may affect fetal oxygenation. Although this may be a useful tool at some point later in this woman's pregnancy, it is not used to diagnose IUGR.

5. The multiple marker test is used to assess the fetus for which condition? a. Down syndrome b. Diaphragmatic hernia c. Congenital cardiac abnormality d. Anencephaly

a. Down syndrome

41. A benign breast condition that includes dilation and inflammation of the collecting ducts is called: a. Ductal ectasia. b. Intraductal papilloma. c. Chronic cystic disease. d. Fibroadenoma.

a. Ductal ectasia.

4. A woman complains of severe abdominal and pelvic pain around the time of menstruation that has gotten worse over the last 5 years. She also complains of pain during intercourse and has tried unsuccessfully to get pregnant for the past 18 months. These symptoms are most likely related to: a. Endometriosis. b. PMS. c. Primary dysmenorrhea. d. Secondary dysmenorrhea.

a. Endometriosis.

The nurse caring for the laboring woman should know that meconium is produced by: a. Fetal intestines. b. Fetal kidneys. c. Amniotic fluid. d. The placenta.

a. Fetal intestines. As the fetus nears term, fetal waste products accumulate in the intestines as dark green-to-black, tarry meconium.

The nurse must be cognizant that an individual's genetic makeup is known as his or her: a. Genotype. b. Phenotype. c. Karyotype. d. Chromotype.

a. Genotype. The genotype comprises all the genes the individual can pass on to a future generation. The phenotype is the observable expression of an individual's genotype. The karyotype is a pictorial analysis of the number, form, and size of an individual's chromosomes. Genotype refers to an individual's genetic makeup.

16. Compared with contraction stress test (CST), nonstress test (NST) for antepartum fetal assessment: a. Has no known contraindications. b. Has fewer false-positive results. c. Is more sensitive in detecting fetal compromise. d. Is slightly more expensive.

a. Has no known contraindications.

25. Which viral sexually transmitted infection is characterized by a primary infection followed by recurrent episodes? a. Herpes simplex virus (HSV)-2 b. Human papillomavirus (HPV) c. Human immunodeficiency virus (HIV) d. Cytomegalovirus (CMV)

a. Herpes simplex virus (HSV)-2

9. From the nurse's perspective, what measure should be the focus of the health care system to reduce the rate of infant mortality further? a. Implementing programs to ensure women's early participation in ongoing prenatal care b. Increasing the length of stay in a hospital after vaginal birth from 2 to 3 days c. Expanding the number of neonatal intensive care units (NICUs) d. Mandating that all pregnant women receive care from an obstetrician

a. Implementing programs to ensure women's early participation in ongoing prenatal care

The nurse caring for a pregnant client knows that her health teaching regarding fetal circulation has been effective when the client reports that she has been sleeping: a. In a side-lying position. b. On her back with a pillow under her knees. c. With the head of the bed elevated. d. On her abdomen.

a. In a side-lying position. Optimal circulation is achieved when the woman is lying at rest on her side. Decreased uterine circulation may lead to intrauterine growth restriction. Previously it was believed that the left lateral position promoted maternal cardiac output, thereby enhancing blood flow to the fetus. However, it is now known that either side-lying position enhances uteroplacental blood flow. If a woman lies on her back with the pressure of the uterus compressing the vena cava, blood return to the right atrium will be diminished. Although this position is recommended and ideal for later in pregnancy, the woman must still maintain a lateral tilt to the pelvis to avoid compression of the vena cava. Many women will find this position uncomfortable as pregnancy advances. Side-lying is the ideal position to promote blood flow to the fetus.

13. Nurses should be aware that the biophysical profile (BPP): a. Is an accurate indicator of impending fetal death. b. Is a compilation of health risk factors of the mother during the later stages of pregnancy. c. Consists of a Doppler blood flow analysis and an amniotic fluid index. d. Involves an invasive form of ultrasound examination.

a. Is an accurate indicator of impending fetal death.

14. As relates to dysfunctional uterine bleeding (DUB), the nurse should be aware that: a. It is most commonly caused by anovulation. b. It most often occurs in middle age. c. The diagnosis of DUB should be the first considered for abnormal menstrual bleeding. d. The most effective medical treatment is steroids.

a. It is most commonly caused by anovulation.

A maternity nurse should be aware of which fact about the amniotic fluid? a. It serves as a source of oral fluid and a repository for waste from the fetus. b. The volume remains about the same throughout the term of a healthy pregnancy. c. A volume of less than 300 ml is associated with gastrointestinal malformations. d. A volume of more than 2 L is associated with fetal renal abnormalities.

a. It serves as a source of oral fluid and a repository for waste from the fetus. Amniotic fluid serves as a source of oral fluid, a repository for waste from the fetus, and also cushions the fetus and helps maintain a constant body temperature. The volume of amniotic fluid changes constantly. Too little amniotic fluid (oligohydramnios) is associated with renal abnormalities. Too much amniotic fluid (hydramnios) is associated with gastrointestinal and other abnormalities.

With regard to prenatal genetic testing, nurses should be aware that: a. Maternal serum screening can determine whether a pregnant woman is at risk of carrying a fetus with Down syndrome. b. Carrier screening tests look for gene mutations of people already showing symptoms of a disease. c. Predisposition testing predicts with near certainty that symptoms will appear. d. Presymptomatic testing is used to predict the likelihood of breast cancer.

a. Maternal serum screening can determine whether a pregnant woman is at risk of carrying a fetus with Down syndrome. Maternal serum screening identifies the risk for the neural tube defect and the specific chromosome abnormality involved in Down syndrome. Carriers of some diseases such as sickle cell disease do not display symptoms. Predisposition testing determines susceptibility such as for breast cancer; presymptomatic testing indicates that, if the gene is present, symptoms are certain to appear.

34. A patient has been prescribed adjuvant tamoxifen therapy. What common side effect might she experience? a. Nausea, hot flashes, and vaginal bleeding b. Vomiting, weight loss, and hair loss c. Nausea, vomiting, and diarrhea d. Hot flashes, weight gain, and headaches

a. Nausea, hot flashes, and vaginal bleeding

9. A woman is undergoing a nipple-stimulated contraction stress test (CST). She is having contractions that occur every 3 minutes. The fetal heart rate (FHR) has a baseline of approximately 120 beats/min without any decelerations. The interpretation of this test is said to be: a. Negative. b. Positive. c. Satisfactory. d. Unsatisfactory.

a. Negative.

35. Despite warnings, prenatal exposure to alcohol continues to exceed by far exposure to illicit drugs. A diagnosis of fetal alcohol syndrome (FAS) is made when there are visible markers in each of three categories. Which is category is not associated with a diagnosis of FAS? a. Respiratory conditions b. Impaired growth c. CNS abnormality d. Craniofacial dysmorphologies

a. Respiratory conditions

13. Which behavior indicates that a woman is "seeking safe passage" for herself and her infant? a. She keeps all prenatal appointments. b. She "eats for two." c. She drives her car slowly. d. She wears only low-heeled shoes.

a. She keeps all prenatal appointments.

29. An essential component of counseling women regarding safe sex practices includes discussion regarding avoiding the exchange of body fluids. The physical barrier promoted for the prevention of sexually transmitted infections and human immunodeficiency virus is the condom. Nurses can help motivate clients to use condoms by initiating a discussion related to a number of aspects of condom use. The most important of these is: a. Strategies to enhance condom use. b. Choice of colors and special features. c. Leaving the decision up to the male partner. d. Places to carry condoms safely.

a. Strategies to enhance condom use.

37. What important, immediate postoperative care practice should the nurse remember when caring for a woman who has had a mastectomy? a. The blood pressure (BP) cuff should not be applied to the affected arm. b. Venipuncture for blood work should be performed on the affected arm. c. The affected arm should be used for intravenous (IV) therapy. d. The affected arm should be held down close to the woman's side.

a. The blood pressure (BP) cuff should not be applied to the affected arm.

15. The transition phase during which ovarian function and hormone production decline is called: a. The climacteric. b. Menarche. c. Menopause. d. Puberty.

a. The climacteric

21. As relates to the father's acceptance of the pregnancy and preparation for childbirth, the maternity nurse should know that: a. The father goes through three phases of acceptance of his own. b. The father's attachment to the fetus cannot be as strong as that of the mother because it does not start until after birth. c. In the last 2 months of pregnancy, most expectant fathers suddenly get very protective of their established lifestyle and resist making changes to the home. d. Typically men remain ambivalent about fatherhood right up to the birth of their child.

a. The father goes through three phases of acceptance of his own.

24. Healthy People 2020 has established national health priorities that focus on a number of maternal-child health indicators. Nurses are assuming greater roles in assessing family health and providing care across the perinatal continuum. Therefore it is important for the nurse to be aware that significant progress has been made in: a. The reduction of fetal deaths and use of prenatal care. b. Low birth weight and preterm birth. c. Elimination of health disparities based on race. d. Infant mortality and the prevention of birth defects.

a. The reduction of fetal deaths and use of prenatal care.

20. The level of practice a reasonably prudent nurse provides is called: a. The standard of care. b. Risk management. c. A sentinel event. d. Failure to rescue.

a. The standard of care.

25. While teaching the expectant mother about personal hygiene during pregnancy, maternity nurses should be aware that: a. Tub bathing is permitted even in late pregnancy unless membranes have ruptured. b. The perineum should be wiped from back to front. c. Bubble bath and bath oils are permissible because they add an extra soothing and cleansing action to the bath. d. Expectant mothers should use specially treated soap to cleanse the nipples.

a. Tub bathing is permitted even in late pregnancy unless membranes have ruptured.

Many parents-to-be have questions about multiple births. Maternity nurses should be able to tell them that: a. Twinning and other multiple births are increasing because of the use of fertility drugs and delayed childbearing. b. Dizygotic twins (two fertilized ova) have the potential to be conjoined twins. c. Identical twins are more common in Caucasian families. d. Fraternal twins are same gender, usually male.

a. Twinning and other multiple births are increasing because of the use of fertility drugs and delayed childbearing. If the parents-to-be are older and have taken fertility drugs, they would be very interested to know about twinning and other multiple births. Conjoined twins are monozygotic; they are from a single fertilized ovum in which division occurred very late. Identical twins show no racial or ethnic preference; fraternal twins are more common among African-American women. Fraternal twins can be different genders or the same gender. Identical twins are the same gender.

2. A 39-year-old primigravida thinks that she is about 8 weeks pregnant, although she has had irregular menstrual periods all her life. She has a history of smoking approximately one pack of cigarettes a day, but she tells you that she is trying to cut down. Her laboratory data are within normal limits. What diagnostic technique could be used with this pregnant woman at this time? a. Ultrasound examination b. Maternal serum alpha-fetoprotein (MSAFP) screening c. Amniocentesis d. Nonstress test (NST)

a. Ultrasound examination

19. The female reproductive organ(s) responsible for cyclic menstruation is/are the: a. Uterus. b. Ovaries. c. Vaginal vestibule. d. Urethra.

a. Uterus.

7. Individual irregularities in the ovarian (menstrual) cycle are most often caused by: a. Variations in the follicular (preovulatory) phase. b. An intact hypothalamic-pituitary feedback mechanism. c. A functioning corpus luteum. d. A prolonged ischemic phase.

a. Variations in the follicular (preovulatory) phase

37. A common effect of both smoking and cocaine use in the pregnant woman is: a. Vasoconstriction b. Increased appetite c. Changes in insulin metabolism d. Increased metabolism

a. Vasoconstriction

14. Through the use of social media technology, nurses can link with other nurses who may share similar interests, insights about practice, and advocate for patients. The most concerning pitfall for nurses using this technology is: a. Violation of patient privacy and confidentiality. b. Institutions and colleagues may be cast in an unfavorable light. c. Unintended negative consequences for using social media. d. Lack of institutional policy governing online contact.

a. Violation of patient privacy and confidentiality.

27. A 25-year-old single woman comes to the gynecologist's office for a follow-up visit related to her abnormal Papanicolaou (Pap) smear. The test revealed that the patient has human papillomavirus (HPV). The client asks, "What is that? Can you get rid of it?" Your best response is: a. "It's just a little lump on your cervix. We can freeze it off." b. "HPV stands for 'human papillomavirus.' It is a sexually transmitted infection (STI) that may lead to cervical cancer." c. "HPV is a type of early human immunodeficiency virus (HIV). You will die from this." d. "You probably caught this from your current boyfriend. He should get tested for this."

b. "HPV stands for 'human papillomavirus.' It is a sexually transmitted infection (STI) that may lead to cervical cancer."

31. Which statement by the patient indicates that she understands breast self-examination? a. "I will examine both breasts in two different positions." b. "I will perform breast self-examination 1 week after my menstrual period starts." c. "I will examine the outer upper area of the breast only." d. "I will use the palm of the hand to perform the examination."

b. "I will perform breast self-examination 1 week after my menstrual period starts."

12. During the first trimester, a woman can expect which of the following changes in her sexual desire? a. An increase, because of enlarging breasts b. A decrease, because of nausea and fatigue c. No change d. An increase, because of increased levels of female hormones

b. A decrease, because of nausea and fatigue

With regard to chromosome abnormalities, nurses should be aware that: a. They occur in approximately 10% of newborns. b. Abnormalities of number are the leading cause of pregnancy loss. c. Down syndrome is a result of an abnormal chromosome structure. d. Unbalanced translocation results in a mild abnormality that the child will outgrow.

b. Abnormalities of number are the leading cause of pregnancy loss. Aneuploidy is an abnormality of number that also is the leading genetic cause of mental retardation. Chromosome abnormalities occur in fewer than 1% of newborns. Down syndrome is the most common form of trisomal abnormality, an abnormality of chromosome number (47 chromosomes). Unbalanced translocation is an abnormality of chromosome structure that often has serious clinical effects.

1. The nurse caring for a newly pregnant woman would advise her that ideally prenatal care should begin: a. Before the first missed menstrual period. b. After the first missed menstrual period. c. After the second missed menstrual period. d. After the third missed menstrual period.

b. After the first missed menstrual period.

6. A woman who is 32 weeks' pregnant is informed by the nurse that a danger sign of pregnancy could be: a. Constipation. b. Alteration in the pattern of fetal movement. c. Heart palpitations. d. Edema in the ankles and feet at the end of the day.

b. Alteration in the pattern of fetal movement.

With regard to the structure and function of the placenta, the maternity nurse should be aware that: a. As the placenta widens, it gradually thins to allow easier passage of air and nutrients. b. As one of its early functions, the placenta acts as an endocrine gland. c. The placenta is able to keep out most potentially toxic substances such as cigarette smoke to which the mother is exposed. d. Optimal blood circulation is achieved through the placenta when the woman is lying on her back or standing.

b. As one of its early functions, the placenta acts as an endocrine gland. The placenta produces four hormones necessary to maintain the pregnancy. The placenta widens until week 20 and continues to grow thicker. Toxic substances such as nicotine and carbon monoxide readily cross the placenta into the fetus. Optimal circulation occurs when the woman is lying on her side.

22. Care management of a woman diagnosed with acute pelvic inflammatory disease (PID) most likely would include: a. Oral antiviral therapy. b. Bed rest in a semi-Fowler position. c. Antibiotic regimen continued until symptoms subside. d. Frequent pelvic examination to monitor the progress of healing.

b. Bed rest in a semi-Fowler position.

4. A 41-week pregnant multigravida presents in the labor and delivery unit after a nonstress test indicated that her fetus could be experiencing some difficulties in utero. Which diagnostic tool would yield more detailed information about the fetus? a. Ultrasound for fetal anomalies b. Biophysical profile (BPP) c. Maternal serum alpha-fetoprotein (MSAFP) screening d. Percutaneous umbilical blood sampling (PUBS)

b. Biophysical profile (BPP)

15. An important development that affects maternity nursing is integrative health care, which: a. Seeks to provide the same health care for all racial and ethnic groups. b. Blends complementary and alternative therapies with conventional Western treatment. c. Focuses on the disease or condition rather than the background of the client. d. Has been mandated by Congress.

b. Blends complementary and alternative therapies with conventional Western treatment.

20. The body part that both protects the pelvic structures and accommodates the growing fetus during pregnancy is the: a. Perineum. b. Bony pelvis. c. Vaginal vestibule. d. Fourchette.

b. Bony pelvis

In presenting to obstetric nurses interested in genetics, the genetic nurse identifies the primary risk(s) associated with genetic testing as: a. Anxiety and altered family relationships. b. Denial of insurance benefits. c. High false positives associated with genetic testing. d. Ethnic and socioeconomic disparity associated with genetic testing.

b. Denial of insurance benefits. Decisions about genetic testing are shaped by socioeconomic status and the ability to pay for the testing. Some types of genetic testing are expensive and are not covered by insurance benefits. Anxiety and altered family relationships, high false positives, and ethnic and socioeconomic disparity are factors that may be difficulties associated with genetic testing, but they are not risks associated with testing.

26. The nurse should have knowledge of the purpose of the pinch test. It is used to: a. Check the sensitivity of the nipples. b. Determine whether the nipple is everted or inverted. c. Calculate the adipose buildup in the abdomen. d. See whether the fetus has become inactive.

b. Determine whether the nipple is everted or inverted.

8. A pregnant woman at 18 weeks of gestation calls the clinic to report that she has been experiencing occasional backaches of mild-to-moderate intensity. The nurse would recommend that she: a. Do Kegel exercises. b. Do pelvic rock exercises. c. Use a softer mattress. d. Stay in bed for 24 hours.

b. Do pelvic rock exercises.

9. During her gynecologic checkup, a 17-year-old girl states that recently she has been experiencing cramping and pain during her menstrual periods. The nurse would document this complaint as: a. Amenorrhea. b. Dysmenorrhea. c. Dyspareunia. d. Premenstrual syndrome (PMS).

b. Dysmenorrhea.

18. A woman has been diagnosed with a high risk pregnancy. She and her husband come into the office in a very anxious state. She seems to be coping by withdrawing from the discussion, showing declining interest. The nurse can best help the couple by: a. Telling her that the physician will isolate the problem with more tests. b. Encouraging her and urging her to continue with childbirth classes. c. Becoming assertive and laying out the decisions the couple needs to make. d. Downplaying her risks by citing success rate studies.

b. Encouraging her and urging her to continue with childbirth classes.

13. During a health history interview, a woman states that she thinks that she has "bumps" on her labia. She also states that she is not sure how to check herself. The correct response would be to: a. Reassure the woman that the examination will not reveal any problems. b. Explain the process of vulvar self-examination to the woman and reassure her that she should become familiar with normal and abnormal findings during the examination. c. Reassure the woman that "bumps" can be treated. d. Reassure her that most women have "bumps" on their labia.

b. Explain the process of vulvar self-examination to the woman and reassure her that she should become familiar with normal and abnormal findings during the examination.

21. To detect human immunodeficiency virus (HIV), most laboratory tests focus on the: a. virus. b. HIV antibodies. c. CD4 counts. d. CD8 counts.

b. HIV antibodies.

28. Which of the following statements about the various forms of hepatitis is accurate? a. A vaccine exists for hepatitis C but not for hepatitis B. b. Hepatitis A is acquired by eating contaminated food or drinking polluted water. c. Hepatitis B is less contagious than human immunodeficiency virus (HIV). d. The incidence of hepatitis C is decreasing.

b. Hepatitis A is acquired by eating contaminated food or drinking polluted water.

26. During which phase of the cycle of violence does the batterer become contrite and remorseful? a. Battering phase b. Honeymoon phase c. Tension-building phase d. Increased drug-taking phase

b. Honeymoon phase

18. The viral sexually transmitted infection (STI) that affects most people in the United States today is: a. Herpes simplex virus type 2 (HSV-2). b. Human papillomavirus (HPV). c. Human immunodeficiency virus (HIV). d. Cytomegalovirus (CMV).

b. Human papillomavirus (HPV).

24. Which nursing intervention is necessary before a second-trimester transabdominal ultrasound? a. Place the woman NPO for 12 hours. b. Instruct the woman to drink 1 to 2 quarts of water. c. Administer an enema. d. Perform an abdominal preparation.

b. Instruct the woman to drink 1 to 2 quarts of water.

You are a maternal-newborn nurse caring for a mother who just delivered a baby born with Down syndrome. What nursing diagnosis would be the most essential in caring for the mother of this infant? a. Disturbed body image b. Interrupted family processes c. Anxiety d. Risk for injury

b. Interrupted family processes This mother likely will experience a disruption in the family process related to the birth of a baby with an inherited disorder. Women commonly experience body image disturbances in the postpartum period, but this is unrelated to giving birth to a child with Down syndrome. The mother likely will have a mix of emotions that may include anxiety, guilt, and denial, but this is not the most essential nursing diagnosis for this family. Risk for injury is not an applicable nursing diagnosis.

19. Maternity nursing care that is based on knowledge gained through research and clinical trials is: a. Derived from the Nursing Intervention Classification. b. Known as evidence-based practice. c. At odds with the Cochrane School of traditional nursing. d. An outgrowth of telemedicine.

b. Known as evidence-based practice.

6. While interviewing a 31-year-old woman before her routine gynecologic examination, the nurse collects data about the client's recent menstrual cycles. The nurse should collect additional information with which statement? a. The woman says her menstrual flow lasts 5 to 6 days. b. She describes her flow as very heavy. c. She reports that she has had a small amount of spotting midway between her periods for the past 2 months. d. She says the length of her menstrual cycle varies from 26 to 29 days.

b. She describes her flow as very heavy.

12. A 62-year-old woman has not been to the clinic for an annual examination for 5 years. The recent death of her husband reminded her that she should come for a visit. Her family doctor has retired, and she is going to see the women's health nurse practitioner for her visit. To facilitate a positive health care experience, the nurse should: a. Remind the woman that she is long overdue for her examination and that she should come in annually. b. Listen carefully and allow extra time for this woman's health history interview. c. Reassure the woman that a nurse practitioner is just as good as her old doctor. d. Encourage the woman to talk about the death of her husband and her fears about her own death.

b. Listen carefully and allow extra time for this woman's health history interview.

12. In the first trimester, ultrasonography can be used to gain information on: a. Amniotic fluid volume. b. Location of Gestational sacs c. Placental location and maturity. d. Cervical length.

b. Location of Gestational sacs

16. What type of cultural concern is the most likely deterrent to many women seeking prenatal care? a. Religion b. Modesty c. Ignorance d. Belief that physicians are evil

b. Modesty

18. In understanding and guiding a woman through her acceptance of pregnancy, a maternity nurse should be aware that: a. Nonacceptance of the pregnancy very often equates to rejection of the child. b. Mood swings most likely are the result of worries about finances and a changed lifestyle as well as profound hormonal changes. c. Ambivalent feelings during pregnancy usually are seen only in emotionally immature or very young mothers. d. Conflicts such as not wanting to be pregnant or childrearing and career-related decisions need not be addressed during pregnancy because they will resolve themselves naturally after birth.

b. Mood swings most likely are the result of worries about finances and a changed lifestyle as well as profound hormonal changes.

9. Physiologically, sexual response can be characterized by: a. Coitus, masturbation, and fantasy. b. Myotonia and vasocongestion. c. Erection and orgasm. d. Excitement, plateau, and orgasm.

b. Myotonia and vasocongestion

20. Risk factors tend to be interrelated and cumulative in their effect. While planning the care for a laboring client with diabetes mellitus, the nurse is aware that she is at a greater risk for: a. Oligohydramnios. b. Polyhydramnios. c. Postterm pregnancy. d. Chromosomal abnormalities.

b. Polyhydramnios.

23. Certain fatty acids classified as hormones that are found in many body tissues and that have roles in many reproductive functions are known as: a. Gonadotropin-releasing hormone (GnRH). b. Prostaglandins (PGs). c. Follicle-stimulating hormone (FSH). d. Luteinizing hormone (LH).

b. Prostaglandins (PGs).

A man's wife is pregnant for the third time. One child was born with cystic fibrosis, and the other child is healthy. The man wonders what the chance is that this child will have cystic fibrosis. This type of testing is known as: a. Occurrence risk. b. Recurrence risk. c. Predictive testing. d. Predisposition testing.

b. Recurrence risk. The couple already has a child with a genetic disease; therefore they will be given a recurrence risk test. If a couple has not yet had children but are known to be at risk for having children with a genetic disease, they are given an occurrence risk test. This couple already has a child with a genetic disorder. Predictive testing is used to clarify the genetic status of an asymptomatic family member. Predisposition testing differs from presymptomatic testing in that a positive result does not indicate 100% risk of a condition developing.

27. A patient at 24 weeks of gestation says she has a glass of wine with dinner every evening. The nurse will counsel her to eliminate all alcohol intake because: a. A daily consumption of alcohol indicates a risk for alcoholism. b. She will be at risk for abusing other substances as well. c. The fetus is placed at risk for altered brain growth. d. The fetus is at risk for multiple organ anomalies.

b. She will be at risk for abusing other substances as well.

2. To ensure optimal outcomes for the patient, the contemporary maternity nurse must incorporate both teamwork and communication with clinicians into her care delivery, The SBAR technique of communication is an easy-to-remember mechanism for communication. Which of the following correctly defines this acronym? a. Situation, baseline assessment, response b. Situation, background, assessment, recommendation c. Subjective background, assessment, recommendation d. Situation, background, anticipated recommendation

b. Situation, background, assessment, recommendation

3. Unique muscle fibers make the uterine myometrium ideally suited for: a. Menstruation. b. The birth process. c. Ovulation. d. Fertilization.

b. The birth process.

21. During a prenatal intake interview, the client informs the nurse that she would prefer a midwife to provide her care during pregnancy and deliver her infant. What information would be most appropriate for the nurse to share with this patient? a. Midwifery care is available only to clients who are uninsured because their services are less expensive than an obstetrician. Costs are often lower than an obstetric provider. b. The client will receive fewer interventions during the birth process. c. The client should be aware that midwives are not certified. d. Delivery can take place only at the client's home or in a birth center.

b. The client will receive fewer interventions during the birth process.

17. A woman who is 6 months pregnant has sought medical attention, saying she fell down the stairs. What scenario would cause an emergency department nurse to suspect that the woman has been a victim of intimate partner violence (IPV)? a. The woman and her partner are having an argument that is loud and hostile. b. The woman has injuries on various parts of her body that are in different stages of healing. c. Examination reveals a fractured arm and fresh bruises. d. She avoids making eye contact and is hesitant to answer questions.

b. The woman has injuries on various parts of her body that are in different stages of healing.

23. With regard to the initial physical examination of a woman beginning prenatal care, maternity nurses should be cognizant of: a. Only women who show physical signs or meet the sociologic profile should be assessed for physical abuse. b. The woman should empty her bladder before the pelvic examination is performed. c. The distribution, amount, and quality of body hair are of no particular importance. d. The size of the uterus is discounted in the initial examination.

b. The woman should empty her bladder before the pelvic examination is performed.

19. The U.S. Centers for Disease Control and Prevention (CDC) recommends that HPV be treated with client-applied: a. Miconazole ointment. b. Topical podofilox 0.5% solution or gel. c. Penicillin given intramuscularly for two doses. d. Metronidazole by mouth.

b. Topical podofilox 0.5% solution or gel.

30. Which statement about multifetal pregnancy is inaccurate? a. The expectant mother often develops anemia because the fetuses have a greater demand for iron. b. Twin pregnancies come to term with the same frequency as single pregnancies. c. The mother should be counseled to increase her nutritional intake and gain more weight. d. Backache and varicose veins often are more pronounced.

b. Twin pregnancies come to term with the same frequency as single pregnancies.

7. Maternal serum alpha-fetoprotein (MSAFP) screening indicates an elevated level. MSAFP screening is repeated and again is reported as higher than normal. What would be the next step in the assessment sequence to determine the well-being of the fetus? a. Percutaneous umbilical blood sampling (PUBS) b. Ultrasound for fetal anomalies c. Biophysical profile (BPP) for fetal well-being d. Amniocentesis for genetic anomalies

b. Ultrasound for fetal anomalies

14. A 3-year-old girl's mother is 6 months pregnant. What concern is this child likely to verbalize? a. How the baby will "get out" b. What the baby will eat c. Whether her mother will die d. What color eyes the baby has

b. What the baby will eat

A pregnant woman at 25 weeks' gestation tells the nurse that she dropped a pan last week and her baby jumped at the noise. Which response by the nurse is most accurate? a. "That must have been a coincidence; babies can't respond like that." b. "The fetus is demonstrating the aural reflex." c. "Babies respond to sound starting at about 24 weeks of gestation." d. "Let me know if it happens again; we need to report that to your midwife."

c. "Babies respond to sound starting at about 24 weeks of gestation." "Babies respond to sound starting at about 24 weeks of gestation" is an accurate statement. "That must have been a coincidence; babies can't respond like that" is inaccurate. Fetuses respond to sound by 24 weeks. Acoustic stimulations can evoke a fetal heart rate response. There is no such thing as an aural reflex. The statement, "Let me know if it happens again; we need to report that to your midwife" is not appropriate; it gives the impression that something is wrong.

30. As a girl progresses through development, she may be at risk for a number of age-related conditions. While preparing a 21-year-old client for her first adult physical examination and Papanicolaou (Pap) test, the nurse is aware of excessiveness shyness. The young woman states that she will not remove her bra because, "There is something wrong with my breasts; one is way bigger." What is the best response by the nurse in this situation? a. "Please reschedule your appointment until you are more prepared." b. "It is okay; the provider will not do a breast examination." c. "I will explain normal growth and breast development to you." d. "That is unfortunate; this must be very stressful for you."

c. "I will explain normal growth and breast development to you."

A couple has been counseled for genetic anomalies. They ask you, "What is karyotyping?" Your best response is: a. "Karyotyping will reveal if the baby's lungs are mature." b. "Karyotyping will reveal if your baby will develop normally." c. "Karyotyping will provide information about the gender of the baby and the number and structure of the chromosomes." d. "Karyotyping will detect any physical deformities the baby has."

c. "Karyotyping will provide information about the gender of the baby and the number and structure of the chromosomes." Karyotyping provides genetic information such as gender and chromosome structure. The L/S, not karyotyping, reveals lung maturity. Although karyotyping can detect genetic anomalies, the range of normal is nondescriptive. Although karyotyping can detect genetic anomalies, not all such anomalies display obvious physical deformities. The term deformities is a nondescriptive word. Furthermore, physical anomalies may be present that are not detected by genetic studies (e.g., cardiac malformations).

18. A 20-year-old patient calls the clinic to report that she has found a lump in her breast. The nurse's best response is: a. "Don't worry about it. I'm sure it's nothing." b. "Wear a tight bra, and it should shrink." c. "Many women have benign lumps and bumps in their breasts. However, to make sure that it's benign, you should come in for an examination by your physician." d. "Check it again in 1 month and call me back if it's still there."

c. "Many women have benign lumps and bumps in their breasts. However, to make sure that it's benign, you should come in for an examination by your physician."

11. The nurse guides a woman to the examination room and asks her to remove her clothes and put on an examination gown with the front open. The woman states, "I have special undergarments that I do not remove for religious reasons." The most appropriate response from the nurse would be: a. "You can't have an examination without removing all your clothes." b. "I'll ask the doctor to modify the examination." c. "Tell me about your undergarments. I'll explain the examination procedure, and then we can discuss how you can have your examination comfortably." d. "What? I've never heard of such a thing! That sounds different and strange."

c. "Tell me about your undergarments. I'll explain the examination procedure, and then we can discuss how you can have your examination comfortably."

Sally comes in for her first prenatal examination. This is her first child. She asks you (the nurse), "How does my baby get air inside my uterus?" The correct response is: a. "The baby's lungs work in utero to exchange oxygen and carbon dioxide." b. "The baby absorbs oxygen from your blood system." c. "The placenta provides oxygen to the baby and excretes carbon dioxide into your bloodstream." d. "The placenta delivers oxygen-rich blood through the umbilical artery to the baby's abdomen."

c. "The placenta provides oxygen to the baby and excretes carbon dioxide into your bloodstream." The placenta functions by supplying oxygen and excreting carbon dioxide to the maternal bloodstream. The fetal lungs do not function for respiratory gas exchange in utero. The baby does not simply absorb oxygen from a woman's blood system. Blood and gas transport occur through the placenta. The placenta delivers oxygen-rich blood through the umbilical vein and not the artery.

A woman's cousin gave birth to an infant with a congenital heart anomaly. The woman asks the nurse when such anomalies occur during development. Which response by the nurse is most accurate? a. "We don't really know when such defects occur." b. "It depends on what caused the defect." c. "They occur between the third and fifth weeks of development." d. "They usually occur in the first 2 weeks of development."

c. "They occur between the third and fifth weeks of development." The cardiovascular system is the first organ system to function in the developing human. Blood vessel and blood formation begins in the third week, and the heart is developmentally complete in the fifth week. "We don't really know when such defects occur" is an inaccurate statement. Regardless of the cause, the heart is vulnerable during its period of development, the third to fifth weeks. "They usually occur in the first 2 weeks of development" is an inaccurate statement.

A father and mother are carriers of phenylketonuria (PKU). Their 2-year-old daughter has PKU. The couple tells the nurse that they are planning to have a second baby. Because their daughter has PKU, they are sure that their next baby won't be affected. What response by the nurse is most accurate? a. "Good planning; you need to take advantage of the odds in your favor." b. "I think you'd better check with your doctor first." c. "You are both carriers, so each baby has a 25% chance of being affected." d. "The ultrasound indicates a boy, and boys are not affected by PKU."

c. "You are both carriers, so each baby has a 25% chance of being affected." The chance is one in four that each child produced by this couple will be affected by PKU disorder. This couple still has an increased likelihood of having a child with PKU. Having one child already with PKU does not guarantee that they will not have another. These parents need to discuss their options with their physician. However, an opportune time has presented itself for the couple to receive correct teaching about inherited genetic risks. No correlation exists between gender and inheritance of the disorder, because PKU is an autosomal recessive disorder.

4. A pregnant woman at 10 weeks of gestation jogs three or four times per week. She is concerned about the effect of exercise on the fetus. The nurse should inform her: a. "You don't need to modify your exercising any time during your pregnancy." b. "Stop exercising because it will harm the fetus." c. "You may find that you need to modify your exercise to walking later in your pregnancy, around the seventh month." d. "Jogging is too hard on your joints; switch to walking now."

c. "You may find that you need to modify your exercise to walking later in your pregnancy, around the seventh month."

23. While working with the pregnant woman in her first trimester, the nurse is aware that chorionic villus sampling (CVS) can be performed during pregnancy at: a. 4 weeks b. 8 weeks c. 10 weeks d. 14 weeks

c. 10 weeks

At approximately _____ weeks of gestation, lecithin is forming on the alveolar surfaces, the eyelids open, and the fetus measures approximately 27 cm crown to rump and weighs approximately 1110 g. a. 20 b. 24 c. 28 d. 30

c. 28 These are all milestones in human development that occur at approximately 28 weeks.

The measurement of lecithin in relation to sphingomyelin (L/S ratio) is used to determine fetal lung maturity. Which ratio reflects maturity of the lungs? a. 1.4:1 b. 1.8:1 c. 2:1 d. 1:1

c. 2:1 A ratio of 2:1 indicates a two-to-one ratio of L/S, an indicator of lung maturity. Ratios of 1.4:1, 1.8:1, and 1:1 indicate immaturity of the fetal lungs.

26. The nurse should know that once human immunodeficiency virus (HIV) enters the body, seroconversion to HIV positivity usually occurs within: a. 6 to 10 days. b. 2 to 4 weeks. c. 6 to 8 weeks. d. 6 months.

c. 6 to 8 weeks.

42. Which patient is most at risk for fibroadenoma of the breast? a. A 38-year-old woman b. A 50-year-old woman c. A 16-year-old girl d. A 27-year-old woman

c. A 16-year-old girl

29. The microscopic examination of scrapings from the cervix, endocervix, or other mucous membranes to detect premalignant or malignant cells is called: a. Bimanual palpation. b. Rectovaginal palpation. c. A Papanicolaou (Pap) test d. A four As procedure.

c. A Papanicolaou (Pap) test

5. At 35 weeks of pregnancy a woman experiences preterm labor. Tocolytics are administered and she is placed on bed rest, but she continues to experience regular uterine contractions, and her cervix is beginning to dilate and efface. What would be an important test for fetal well-being at this time? a. Percutaneous umbilical blood sampling (PUBS) b. Ultrasound for fetal size c. Amniocentesis for fetal lung maturity d. Nonstress test (NST)

c. Amniocentesis for fetal lung maturity

33. A woman who is older than 35 years may have difficulty achieving pregnancy primarily because: a. Personal risk behaviors influence fertility b. She has used contraceptives for an extended time c. Her ovaries may be affected by the aging process d. Prepregnancy medical attention is lacking

c. Her ovaries may be affected by the aging process

14. A woman arrives at the clinic for her annual examination. She tells the nurse that she thinks she has a vaginal infection and she has been using an over-the-counter cream for the past 2 days to treat it. The nurse's initial response should be to: a. Inform the woman that vaginal creams may interfere with the Papanicolaou (Pap) test for which she is scheduled. b. Reassure the woman that using vaginal cream is not a problem for the examination. c. Ask the woman to describe the symptoms that indicate to her that she has a vaginal infection. d. Ask the woman to reschedule the appointment for the examination.

c. Ask the woman to describe the symptoms that indicate to her that she has a vaginal infection.

16. Recent trends in childbirth practices in the United States indicate that: a. More than 15% of mothers had late or no prenatal care. b. The percentage of Hispanics, non-Hispanic African Americans, and Caucasians who received prenatal care was essentially the same. c. Births occurring in the hospital accounted for 99% of births. d. Cesarean births have been declining as a percentage of live births.

c. Births occurring in the hospital accounted for 99% of births.

43. The drug of choice for treatment of gonorrhea is: a. Penicillin G. b. Tetracycline. c. Ceftriaxone. d. Acyclovir.

c. Ceftriaxone.

17. When evaluating a patient for sexually transmitted infections (STIs), the nurse should be aware that the most common bacterial STI is: a. Gonorrhea. b. Syphilis. c. Chlamydia. d. Candidiasis.

c. Chlamydia.

The nurse is providing genetic counseling for an expectant couple who already have a child with trisomy 18. The nurse should: a. Tell the couple they need to have an abortion within 2 to 3 weeks. b. Explain that the fetus has a 50% chance of having the disorder. c. Discuss options with the couple, including amniocentesis to determine whether the fetus is affected. d. Refer the couple to a psychologist for emotional support.

c. Discuss options with the couple, including amniocentesis to determine whether the fetus is affected. Genetic testing, including amniocentesis, would need to be performed to determine whether the fetus is affected. The couple should be given information about the likelihood of having another baby with this disorder so that they can make an informed decision. A genetic counselor is the best source for determining genetic probability ratios. The couple eventually may need emotional support, but the status of the pregnancy must be determined first.

24. With regard to follow-up visits for women receiving prenatal care, nurses should be aware that: a. The interview portions become more intensive as the visits become more frequent over the course of the pregnancy. b. Monthly visits are scheduled for the first trimester, every 2 weeks for the second trimester, and weekly for the third trimester. c. During the abdominal examination, the nurse should be alert for supine hypotension. d. For pregnant women, a systolic blood pressure (BP) of 130 and a diastolic BP of 80 is sufficient to be considered hypertensive.

c. During the abdominal examination, the nurse should be alert for supine hypotension.

32. In response to requests by the U.S. Public Health Service for new models of prenatal care, an innovative new approach to prenatal care known as centering pregnancy was developed. Which statement would accurately apply to the centering model of care? a. Group sessions begin with the first prenatal visit. b. At each visit, blood pressure, weight, and urine dipsticks are obtained by the nurse. c. Eight to 12 women are placed in gestational-age cohort groups. d. Outcomes are similar to those of traditional prenatal care.

c. Eight to 12 women are placed in gestational-age cohort groups.

31. Which diagnostic test is used to confirm a suspected diagnosis of breast cancer? a. Mammogram b. Ultrasound c. Fine-needle aspiration (FNA) d. CA 15.3

c. Fine-needle aspiration (FNA)

19. In the past, factors to determine whether a woman was likely to develop a high risk pregnancy were evaluated primarily from a medical point of view. A broader, more comprehensive approach to high-risk pregnancy has been adopted today. There are now four categories based on threats to the health of the woman and the outcome of pregnancy. Which of the following is not one of these categories? a. Biophysical b. Psychosocial c. Geographic d. Environmental

c. Geographic

27. To provide the patient with accurate information about dental care during pregnancy, maternity nurses should be aware that: a. Dental care can be dropped from the priority list because the woman has enough to worry about and is getting a lot of calcium anyway. b. Dental surgery, in particular, is contraindicated because of the psychologic stress it engenders. c. If dental treatment is necessary, the woman will be most comfortable with it in the second trimester. d. Dental care interferes with the expectant mother's need to practice conscious relaxation.

c. If dental treatment is necessary, the woman will be most comfortable with it in the second trimester.

23. To ensure patient safety, the practicing nurse must have knowledge of the current Joint Commission's "Do Not Use" list of abbreviations. Which of the following is acceptable for use? a. q.o.d. or Q.O.D. b. MSO4 or MgSO4 c. International Unit d. Lack of a leading zero

c. International Unit

38. A woman has a breast mass that is not well delineated and is nonpalpable, immobile, and nontender. This is most likely: a. Fibroadenoma. b. Lipoma. c. Intraductal papilloma. d. Mammary duct ectasia.

c. Intraductal papilloma.

17. The nurse providing care for the antepartum woman should understand that contraction stress test (CST): a. Sometimes uses vibroacoustic stimulation. b. Is an invasive test; however, contractions are stimulated. c. Is considered negative if no late decelerations are observed with the contractions. d. Is more effective than nonstress test (NST) if the membranes have already been ruptured.

c. Is considered negative if no late decelerations are observed with the contractions.

12. With regard to endometriosis, nurses should be aware that: a. It is characterized by the presence and growth of endometrial tissue inside the uterus. b. It is found more often in African-American women than in white or Asian women. c. It may worsen with repeated cycles or remain asymptomatic and disappear after menopause. d. It is unlikely to affect sexual intercourse or fertility.

c. It may worsen with repeated cycles or remain asymptomatic and disappear after menopause.

1. When assessing a patient for amenorrhea, the nurse should be aware that this is unlikely to be caused by: a. Anatomic abnormalities. b. Type 1 diabetes mellitus. c. Lack of exercise. d. Hysterectomy.

c. Lack of exercise.

44. The nurse providing education regarding breast care should explain to the woman that fibrocystic changes in breasts are: a. A disease of the milk ducts and glands in the breasts. b. A premalignant disorder characterized by lumps found in the breast tissue. c. Lumpiness with pain and tenderness found in varying degrees in the breast tissue of healthy women during menstrual cycles. d. Lumpiness accompanied by tenderness after menses.

c. Lumpiness with pain and tenderness found in varying degrees in the breast tissue of healthy women during menstrual cycles

With regard to the development of the respiratory system, maternity nurses should be aware that: a. The respiratory system does not begin developing until after the embryonic stage. b. The infant's lungs are considered mature when the lecithin/sphingomyelin (L/S) ratio is 1:1, at about 32 weeks. c. Maternal hypertension can reduce maternal-placental blood flow, accelerating lung maturity. d. Fetal respiratory movements are not visible on ultrasound scans until at least 16 weeks.

c. Maternal hypertension can reduce maternal-placental blood flow, accelerating lung maturity. A reduction in placental blood flow stresses the fetus, increases blood levels of corticosteroids, and thus accelerates lung maturity. Development of the respiratory system begins during the embryonic phase and continues into childhood. The infant's lungs are mature when the L/S ratio is 2:1, at about 35 weeks. Lung movements have been seen on ultrasound scans at 11 weeks.

8. A 36-year-old woman has been given a diagnosis of uterine fibroids. When planning care for this patient, the nurse should know that: a. Fibroids are malignant tumors of the uterus that require radiation or chemotherapy. b. Fibroids increase in size during the perimenopausal period. c. Menorrhagia is a common finding. d. The woman is unlikely to become pregnant as long as the fibroids are in her uterus.

c. Menorrhagia is a common finding.

8. Prostaglandins are produced in most organs of the body, including the uterus. Other source(s) of prostaglandins is/are: a. Ovaries. b. Breast milk. c. Menstrual blood. d. The vagina.

c. Menstrual blood.

15. Management of primary dysmenorrhea often requires a multifaceted approach. The nurse who provides care for a client with this condition should be aware that the optimal pharmacologic therapy for pain relief is: a. Acetaminophen. b. Oral contraceptives (OCPs). c. Nonsteroidal antiinflammatory drugs (NSAIDs). d. Aspirin.

c. Nonsteroidal antiinflammatory drugs (NSAIDs).

22. With regard to the initial visit with a client who is beginning prenatal care, nurses should be aware that: a. The first interview is a relaxed, get-acquainted affair in which nurses gather some general impressions. b. If nurses observe handicapping conditions, they should be sensitive and not inquire about them because the client will do that in her own time. c. Nurses should be alert to the appearance of potential parenting problems, such as depression or lack of family support. d. Because of legal complications, nurses should not ask about illegal drug use; that is left to physicians.

c. Nurses should be alert to the appearance of potential parenting problems, such as depression or lack of family support.

3. The role of the professional nurse caring for childbearing families has evolved to emphasize: a. Providing care to patients directly at the bedside. b. Primarily hospital care of maternity patients. c. Practice using an evidence-based approach. d. Planning patient care to cover longer hospital stays.

c. Practice using an evidence-based approach.

7. When evaluating a patient whose primary complaint is amenorrhea, the nurse must be aware that lack of menstruation is most often the result of: a. Stress. b. Excessive exercise. c. Pregnancy. d. Eating disorders.

c. Pregnancy.

11. Which statement concerning cyclic perimenstrual pain and discomfort (CPPD) is accurate? a. Premenstrual dysphoric disorder (PMDD) is a milder form of premenstrual syndrome (PMS) and more common in younger women. b. Secondary dysmenorrhea is more intense and medically significant than primary dysmenorrhea. c. Premenstrual syndrome is a complex, poorly understood condition that may include any of a hundred symptoms. d. The causes of PMS have been well established.

c. Premenstrual syndrome is a complex, poorly understood condition that may include any of a hundred symptoms.

10. Alternative and complementary therapies: a. Replace conventional Western modalities of treatment. b. Are used by only a small number of American adults. c. Recognize the value of clients' input into their health care. d. Focus primarily on the disease an individual is experiencing.

c. Recognize the value of clients' input into their health care.

21. A fully matured endometrium that has reached the thickness of heavy, soft velvet describes the _____ phase of the endometrial cycle. a. Menstrual b. Proliferative c. Secretory d. Ischemic

c. Secretory

32. A pregnant woman who abuses cocaine admits to exchanging sex for her drug habit. This behavior places her at a greater risk for: a. Depression of the central nervous system b. Hypotension and vasodilation c. Sexually transmitted diseases d. Postmature birth

c. Sexually transmitted diseases

11. The nurse should be aware that the partner's main role in pregnancy is to: a. Provide financial support. b. Protect the pregnant woman from "old wives' tales." c. Support and nurture the pregnant woman. d. Make sure the pregnant woman keeps prenatal appointments.

c. Support and nurture the pregnant woman.

6. Menstruation is periodic uterine bleeding: a. That occurs every 28 days. b. In which the entire uterine lining is shed. c. That is regulated by ovarian hormones. d. That leads to fertilization.

c. That is regulated by ovarian hormones.

10. When nurses help their expectant mothers assess the daily fetal movement counts, they should be aware that: a. Alcohol or cigarette smoke can irritate the fetus into greater activity. b. "Kick counts" should be taken every half hour and averaged every 6 hours, with every other 6-hour stretch off. c. The fetal alarm signal should go off when fetal movements stop entirely for 12 hours. d. Obese mothers familiar with their bodies can assess fetal movement as well as average-size women.

c. The fetal alarm signal should go off when fetal movements stop entirely for 12 hours.

In practical terms regarding genetic health care, nurses should be aware that: a. Genetic disorders affect equally people of all socioeconomic backgrounds, races, and ethnic groups. b. Genetic health care is more concerned with populations than individuals. c. The most important of all nursing functions is providing emotional support to the family during counseling. d. Taking genetic histories is the province of large universities and medical centers.

c. The most important of all nursing functions is providing emotional support to the family during counseling. Nurses should be prepared to help with a variety of stress reactions from a couple facing the possibility of a genetic disorder. Although anyone may have a genetic disorder, certain disorders appear more often in certain ethnic and racial groups. Genetic health care is highly individualized because treatments are based on the phenotypic responses of the individual. Individual nurses at any facility can take a genetic history, although larger facilities may have better support services.

24. Which statement regarding female sexual response is inaccurate? a. Women and men are more alike than different in their physiologic response to sexual arousal and orgasm. b. Vasocongestion is the congestion of blood vessels. c. The orgasmic phase is the final state of the sexual response cycle. d. Facial grimaces and spasms of hands and feet are often part of arousal.

c. The orgasmic phase is the final state of the sexual response cycle.

14. With regard to amniocentesis, nurses should be aware that: a. Because of new imaging techniques, amniocentesis is now possible in the first trimester. b. Despite the use of ultrasound, complications still occur in the mother or infant in 5% to 10% of cases. c. The shake test, or bubble stability test, is a quick means of determining fetal maturity. d. The presence of meconium in the amniotic fluid is always cause for concern.

c. The shake test, or bubble stability test, is a quick means of determining fetal maturity.

11. A 38-year-old Hispanic woman delivered a 9-pound, 6-ounce girl vaginally after being in labor for 43 hours. The baby died 3 days later from sepsis. On what grounds would the woman potentially have a legitimate legal case for negligence? a. She is Hispanic. b. She delivered a girl. c. The standards of care were not met. d. She refused fetal monitoring.

c. The standards of care were not met.

35. After a mastectomy a woman should be instructed to perform all of the following except: a. Emptying surgical drains twice a day and as needed. b. Avoiding lifting more than 4.5 kg (10 lb) or reaching above her head until given permission by her surgeon. c. Wearing clothing with snug sleeves to support the tissue of the arm on the operative side. d. Reporting immediately if inflammation develops at the incision site or in the affected arm.

c. Wearing clothing with snug sleeves to support the tissue of the arm on the operative side.

19. With regard to a woman's reordering of personal relationships during pregnancy, the maternity nurse should understand that: a. Because of the special motherhood bond, a woman's relationship with her mother is even more important than with the father of the child. b. Nurses need not get involved in any sexual issues the couple has during pregnancy, particularly if they have trouble communicating them to each other. c. Women usually express two major relationship needs during pregnancy: feeling loved and valued and having the child accepted by the father. d. The woman's sexual desire is likely to be highest in the first trimester because of the excitement and because intercourse is physically easier.

c. Women usually express two major relationship needs during pregnancy: feeling loved and valued and having the child accepted by the father.

7. A woman who is 14 weeks pregnant tells the nurse that she always had a glass of wine with dinner before she became pregnant. She has abstained during her first trimester and would like to know if it is safe for her to have a drink with dinner now. The nurse would tell her: a. "Since you're in your second trimester, there's no problem with having one drink with dinner." b. "One drink every night is too much. One drink three times a week should be fine." c. "Since you're in your second trimester, you can drink as much as you like." d. "Because no one knows how much or how little alcohol it takes to cause fetal problems, the best course is to abstain throughout your pregnancy."

d. "Because no one knows how much or how little alcohol it takes to cause fetal problems, the best course is to abstain throughout your pregnancy."

10. A woman is 3 months pregnant. At her prenatal visit, she tells the nurse that she doesn't know what is happening; one minute she's happy that she is pregnant, and the next minute she cries for no reason. Which response by the nurse is most appropriate? a. "Don't worry about it; you'll feel better in a month or so." b. "Have you talked to your husband about how you feel?" c. "Perhaps you really don't want to be pregnant." d. "Hormonal changes during pregnancy commonly result in mood swings."

d. "Hormonal changes during pregnancy commonly result in mood swings."

20. What represents a typical progression through the phases of a woman's establishing a relationship with the fetus? a. Accepts the fetus as distinct from herself—accepts the biologic fact of pregnancy—has a feeling of caring and responsibility b. Fantasizes about the child's gender and personality—views the child as part of herself—becomes introspective c. Views the child as part of herself—has feelings of well-being—accepts the biologic fact of pregnancy d. "I am pregnant."—"I am going to have a baby."—"I am going to be a mother."

d. "I am pregnant."—"I am going to have a baby."—"I am going to be a mother."

22. While obtaining a detailed history from a woman who has recently emigrated from Somalia, the nurse realizes that the client has undergone female genital mutilation (FGM). The nurse's best response to this patient is: a. "This is a very abnormal practice and rarely seen in the United States." b. "Do you know who performed this so that it can be reported to the authorities?" c. "We will be able to restore your circumcision fully after delivery." d. "The extent of your circumcision will affect the potential for complications."

d. "The extent of your circumcision will affect the potential for complications."

32. A healthy 60-year-old African-American woman regularly receives her health care at the clinic in her neighborhood. She is due for a mammogram. At her previous clinic visit, her physician, concerned about the 3-week wait at the neighborhood clinic, made an appointment for her to have a mammogram at a teaching hospital across town. She did not keep her appointment and returned to the clinic today to have the nurse check her blood pressure. What would be the most appropriate statement for the nurse to make to this patient? a. "Do you have transportation to the teaching hospital so that you can get your mammogram?" b. "I'm concerned that you missed your appointment; let me make another one for you." c. "It's very dangerous to skip your mammograms; your breasts need to be checked." d. "Would you like me to make an appointment for you to have your mammogram here?"

d. "Would you like me to make an appointment for you to have your mammogram here?"

A woman who is 8 months pregnant asks the nurse, "Does my baby have any antibodies to fight infection?" The most appropriate response by the nurse is: a. "Your baby has all the immune globulins (Ig) necessary: IgG, IgM, and IgA." b. "Your baby won't receive any antibodies until he is born and you breastfeed him." c. "Your baby does not have any antibodies to fight infection." d. "Your baby has IgG and IgM."

d. "Your baby has IgG and IgM." During the third trimester the only Ig that crosses the placenta, IgG, provides passive acquired immunity to specific bacterial toxins. The fetus produces IgM by the end of the first trimester. IgAs are not produced by the baby. By the third trimester the fetus has IgG and IgM. Breastfeeding supplies the baby with IgA. "Your baby does not have any antibodies to fight infection" is not an accurate statement.

36. A nurse practitioner performs a clinical breast examination on a woman diagnosed with fibroadenoma. The nurse knows that fibroadenoma is characterized by: a. Inflammation of the milk ducts and glands behind the nipples. b. Thick, sticky discharge from the nipple of the affected breast. c. Lumpiness in both breasts that develops 1 week before menstruation. d. A single lump in one breast that can be expected to shrink as the woman ages.

d. A single lump in one breast that can be expected to shrink as the woman ages.

13. The term used to describe legal and professional responsibility for practice for maternity nurses is: a. Collegiality. b. Ethics. c. Evaluation. d. Accountability.

d. Accountability.

5. During a prenatal intake interview, the nurse is in the process of obtaining an initial assessment of a 21-year-old Hispanic patient with limited English proficiency. It is important for the nurse to: a. Use maternity jargon in order for the patient to become familiar with these terms. b. Speak quickly and efficiently to expedite the visit. c. Provide the patient with handouts. d. Assess whether the patient understands the discussion.

d. Assess whether the patient understands the discussion.

5. Nafarelin is currently used as a treatment for mild-to-severe endometriosis. The nurse should tell a woman taking this medication that the drug: a. Stimulates the secretion of gonadotropin-releasing hormone (GnRH), thereby stimulating ovarian activity. b. Should be sprayed into one nostril every other day. c. Should be injected into subcutaneous tissue BID. d. Can cause her to experience some hot flashes and vaginal dryness.

d. Can cause her to experience some hot flashes and vaginal dryness.

40. The nurse providing care in a women's health care setting must be aware regarding which sexually transmitted infection that can be successfully treated and cured? a. Herpes b. Acquired immunodeficiency syndrome (AIDS) c. Venereal warts d. Chlamydia

d. Chlamydia

The _____ is/are responsible for oxygen and carbon dioxide transport to and from the maternal bloodstream. a. Decidua basalis b. Blastocyst c. Germ layer d. Chorionic villi

d. Chorionic villi Chorionic villi are fingerlike projections that develop out of the trophoblast and extend into the blood-filled spaces of the endometrium. The villi obtain oxygen and nutrients from the maternal bloodstream and dispose of carbon dioxide and waste products into the maternal blood. The decidua basalis is the portion of the decidua (endometrium) under the blastocyst where the villi attach. The blastocyst is the embryonic development stage after the morula. Implantation occurs at this stage. The germ layer is a layer of the blastocyst.

28. As a powerful central nervous system stimulant, which of these substances can lead to miscarriage, preterm labor, placental separation (abruption), and stillbirth? a. Heroin b. Alcohol c. PCP d. Cocaine

d. Cocaine

8. When the nurse is unsure about how to perform a patient care procedure, the best action would be to: a. Ask another nurse. b. Discuss the procedure with the patient's physician. c. Look up the procedure in a nursing textbook. d. Consult the agency procedure manual and follow the guidelines for the procedure.

d. Consult the agency procedure manual and follow the guidelines for the procedure.

33. The nurse must watch for what common complications in a patient who has undergone a transverse rectus abdominis myocutaneous (TRAM) flap? a. Axillary edema and tissue necrosis b. Delayed wound healing and muscle contractions c. Delayed wound healing and axillary edema d. Delayed wound healing and hematoma

d. Delayed wound healing and hematoma

31. The phenomenon of someone other than the mother-to-be experiencing pregnancy-like symptoms such as nausea and weight gain applies to the: a. Mother of the pregnant woman. b. Couple's teenage daughter. c. Sister of the pregnant woman. d. Expectant father.

d. Expectant father.

4. A 23-year-old African-American woman is pregnant with her first child. Based on the statistics for infant mortality, which plan is most important for the nurse to implement? a. Perform a nutrition assessment. b. Refer the woman to a social worker. c. Advise the woman to see an obstetrician, not a midwife. d. Explain to the woman the importance of keeping her prenatal care appointments.

d. Explain to the woman the importance of keeping her prenatal care appointments.

1. A woman arrives at the clinic seeking confirmation that she is pregnant. The following information is obtained: She is 24 years old with a body mass index (BMI) of 17.5. She admits to having used cocaine "several times" during the past year and drinks alcohol occasionally. Her blood pressure (BP) is 108/70 mm Hg, her pulse rate is 72 beats/min, and her respiratory rate is 16 breaths/min. The family history is positive for diabetes mellitus and cancer. Her sister recently gave birth to an infant with a neural tube defect (NTD). Which characteristics place the woman in a high risk category? a. Blood pressure, age, BMI b. Drug/alcohol use, age, family history c. Family history, blood pressure, BMI d. Family history, BMI, drug/alcohol abuse

d. Family history, BMI, drug/alcohol abuse

33. While you are assessing the vital signs of a pregnant woman in her third trimester, the patient complains of feeling faint, dizzy, and agitated. Which nursing intervention is appropriate? a. Have the patient stand up and retake her blood pressure. b. Have the patient sit down and hold her arm in a dependent position. c. Have the patient lie supine for 5 minutes and recheck her blood pressure on both arms. d. Have the patient turn to her left side and recheck her blood pressure in 5 minutes.

d. Have the patient turn to her left side and recheck her blood pressure in 5 minutes.

25. As part of their participation in the gynecologic portion of the physical examination, nurses should: a. Take a firm approach that encourages the client to facilitate the examination by following the physician's instructions exactly. b. Explain the procedure as it unfolds and continue to question the client to get information in a timely manner. c. Take the opportunity to explain that the trendy vulvar self-examination is only for women at risk for cancer. d. Help the woman relax through proper placement of her hands and proper breathing during the examination.

d. Help the woman relax through proper placement of her hands and proper breathing during the examination.

9. For what reason would breastfeeding be contraindicated? a. Hepatitis B b. Everted nipples c. History of breast cancer 3 years ago d. Human immunodeficiency virus (HIV) positive

d. Human immunodeficiency virus (HIV) positive

22. The stimulated release of gonadotropin-releasing hormone and follicle-stimulating hormone is part of the: a. Menstrual cycle. c. Ovarian cycle. b. Endometrial cycle. c. Ovarian cycle. d. Hypothalamic-pituitary cycle.

d. Hypothalamic-pituitary cycle.

The most basic information a maternity nurse should have concerning conception is that: a. Ova are considered fertile 48 to 72 hours after ovulation. b. Sperm remain viable in the woman's reproductive system for an average of 12 to 24 hours. c. Conception is achieved when a sperm successfully penetrates the membrane surrounding the ovum. d. Implantation in the endometrium occurs 6 to 10 days after conception.

d. Implantation in the endometrium occurs 6 to 10 days after conception. After implantation, the endometrium is called the decidua. Ova are considered fertile for about 24 hours after ovulation. Sperm remain viable in the woman's reproductive system for an average of 2 to 3 days. Penetration of the ovum by the sperm is called fertilization. Conception occurs when the zygote, the first cell of the new individual, is formed.

6. When managing health care for pregnant women at a prenatal clinic, the nurse should recognize that the most significant barrier to access to care is the pregnant woman's: a. Age. b. Minority status. c. Educational level. d. Inability to pay.

d. Inability to pay.

34. The most dangerous effect on the fetus of a mother who smokes cigarettes while pregnant is: a. Genetic changes and anomalies b. Extensive central nervous system damage c. Fetal addiction to the substance inhaled d. Intrauterine growth restriction

d. Intrauterine growth restriction

13. One of the alterations in cyclic bleeding that occurs between periods is called: a. Oligomenorrhea. b. Menorrhagia. c. Leiomyoma. d. Metrorrhagia.

d. Metrorrhagia.

17. Recent trends in childbirth practice indicate that: a. Delayed pushing is now discouraged in the second stage of labor. b. Episiotomy rates are increasing. c. Midwives perform more episiotomies than physicians. d. Newborn infants remain with the mother and are encouraged to breastfeed.

d. Newborn infants remain with the mother and are encouraged to breastfeed.

1. The two primary functions of the ovary are: a. Normal female development and sex hormone release. b. Ovulation and internal pelvic support. c. Sexual response and ovulation. d. Ovulation and hormone production.

d. Ovulation and hormone production.

10. With regard to dysmenorrhea, nurses should be aware that: a. It is more common in older women. b. It is more common in leaner women who exercise strenuously. c. Symptoms can begin at any point in the ovulatory cycle. d. Pain usually occurs in the suprapubic area or lower abdomen.

d. Pain usually occurs in the suprapubic area or lower abdomen.

4. The hormone responsible for maturation of mammary gland tissue is: a. Estrogen. b. Testosterone. c. Prolactin. d. Progesterone.

d. Progesterone.

25. The nurse recognizes that a nonstress test (NST) in which two or more fetal heart rate (FHR) accelerations of 15 beats/min or more occur with fetal movement in a 20-minute period is: a. Nonreactive b. Positive c. Negative d. Reactive

d. Reactive

10. The long-term treatment plan for an adolescent with an eating disorder focuses on: a. Managing the effects of malnutrition. b. Establishing sufficient caloric intake. c. Improving family dynamics. d. Restructuring perception of body image.

d. Restructuring perception of body image.

16. The two primary areas of risk for sexually transmitted infections (STIs) are: a. Sexual orientation and socioeconomic status. b. Age and educational level. c. Large number of sexual partners and race. d. Risky sexual behaviors and inadequate preventive health behaviors.

d. Risky sexual behaviors and inadequate preventive health behaviors.

30. The nurse who is teaching a group of women about breast cancer would tell the women that: a. Risk factors identify more than 50% of women who will develop breast cancer. b. Nearly 90% of lumps found by women are malignant. c. One in 10 women in the United States will develop breast cancer in her lifetime. d. The exact cause of breast cancer is unknown.

d. The exact cause of breast cancer is unknown.

With regard to the estimation and interpretation of the recurrence of risks for genetic disorders, nurses should be aware that: a. With a dominant disorder the likelihood of the second child also having the condition is 100%. b. An autosomal recessive disease carries a one in eight risk of the second child also having the disorder. c. Disorders involving maternal ingestion of drugs carry a one in four chance of being repeated in the second child. d. The risk factor remains the same no matter how many affected children are already in the family.

d. The risk factor remains the same no matter how many affected children are already in the family. Each pregnancy is an independent event. The risk factor (e.g., one in two, one in four) remains the same for each child, no matter how many children are born to the family. In a dominant disorder the likelihood of recurrence in subsequent children is 50% (one in two). An autosomal recessive disease carries a one in four chance of recurrence. Subsequent children would be at risk only if the mother continued to take drugs; the rate of risk would be difficult to calculate.

11. In comparing the abdominal and transvaginal methods of ultrasound examination, nurses should explain to their clients that: a. Both require the woman to have a full bladder. b. The abdominal examination is more useful in the first trimester. c. Initially the transvaginal examination can be painful. d. The transvaginal examination allows pelvic anatomy to be evaluated in greater detail.

d. The transvaginal examination allows pelvic anatomy to be evaluated in greater detail.

15. In her work with pregnant women of various cultures, a nurse practitioner has observed various practices that seemed strange or unusual. She has learned that cultural rituals and practices during pregnancy seem to have one purpose in common. Which statement best describes that purpose? a. To promote family unity b. To ward off the "evil eye" c. To appease the gods of fertility d. To protect the mother and fetus during pregnancy

d. To protect the mother and fetus during pregnancy

6. A 40-year-old woman with a high body mass index (BMI) is 10 weeks pregnant. Which diagnostic tool is appropriate to suggest to her at this time? A. Biophysical profile B. Amniocentesis C. Maternal serum alpha-fetoprotein (MSAFP) D. Transvaginal ultrasound

d. Transvaginal ultrasound

2. When a nurse is counseling a woman for primary dysmenorrhea, which nonpharmacologic intervention might be recommended? a. Increasing the intake of red meat and simple carbohydrates b. Reducing the intake of diuretic foods such as peaches and asparagus c. Temporarily substituting physical activity for a sedentary lifestyle d. Using a heating pad on the abdomen to relieve cramping

d. Using a heating pad on the abdomen to relieve cramping

3. Which symptom is considered a first-trimester warning sign and should be reported immediately by the pregnant woman to her health care provider? a. Nausea with occasional vomiting b. Fatigue c. Urinary frequency d. Vaginal bleeding

d. Vaginal bleeding

28. When discussing work and travel during pregnancy with a pregnant patient, nurses should instruct them that: a. Women should sit for as long as possible and cross their legs at the knees from time to time for exercise. b. Women should avoid seat belts and shoulder restraints in the car because they press on the fetus. c. Metal detectors at airport security checkpoints can harm the fetus if the woman passes through them a number of times. d. While working or traveling in a car or on a plane, women should arrange to walk around at least every hour or so.

d. While working or traveling in a car or on a plane, women should arrange to walk around at least every hour or so.

24. The recommended treatment for the prevention of human immunodeficiency virus (HIV) transmission to the fetus during pregnancy is: a. Acyclovir. b. Ofloxacin. c. Podophyllin. d. Zidovudine.

d. Zidovudine.


Related study sets

Lugares en la ciudad español 2 unidad 1

View Set

Google Data Analytics Professional

View Set

ISDS 351 (CHAP 11: Project Risk Management)

View Set

Chapter 12: Reports on Audited Financial Statements (Connect SB)

View Set

AP World History - Chapters 15, 18, 19, and 20 Vocabulary

View Set

Greek Terms I Should Probably Know

View Set